Sei sulla pagina 1di 57

A1 PASSERS TRAINING, RESEARCH, REVIEW AND DEVELOPMENT COMPANY

FINAL COMPREHENSIVE EXAMINATIONS


NURSING PRACTICE I-Basic Foundation of Nursing and Professional Nursing Practice
GENERAL INSTRUCTIONS:
1. This test booklet contains 100 test questions.
2. Read INSTRUCTIONS TO EXAMINEES printed on your answer sheet.
3. Shade only one (1) box for each question on your answer sheets. Two or more boxes shaded will invalidate
your answer.
4. AVOID ERASURES.
INSTRUCTIONS:
1. Detach one (1) answer sheet from the bottom of your Examinee ID/Answer Sheet Set.
2. Write the subject title Nursing Practice I on the box provided.
3. Shade Set Box A on your answer sheet if your test booklet is Set A; Set Box B if your test booklet is Set B.
__________________________________________________________________________________________

Situation 1: Taking care of clients is challenging Mrs. Marina, 65 year old client is terminally ill with
stage 4 cancer of the breast, right with metastasis to the cervical spine and lungs. She is ambulant but
weak, able to perform her activities of daily living and has no complaints of pain.
1. You are assigned to take care Mrs. Marina. She does not ask questions regarding her condition and her
relatives do not know how much knowledge she has about her prognosis. Considering the clients
needs and to provide holistic care to the client, your plan of care should focus on:
a. Continuous assessment of her condition to keep her clean and comfortable
b. Providing support to the client as she ambulate in the room
c. Ensuring that her physiologic needs are met especially nutrition
d. Assessing the clients percaption of her illness and thoughts about dying
2. During the physicians visit where he discussed the clients condition with Mrs. Marinas children, the
client overhead part of the conversation where the doctor said, she has short time left to live. After the
physician left the client said to the nurse Why didnt anyone tell me? Im not ready to die. What would
be your appropriate response?
a. This is time must be very difficult for you
b. It is any consolation, everyone has to die sooner or later
c. Death and birth are normal parts of the cycle of life
d. You will be fine, you are in good hands.
3. Mrs. Marina conditions: worsens. She has begun to experience severe pain and manifest signs of
death. The children ask you if their mother is going to die soon. Which of the following is your MOST
APPROPRIATE response?
a. The signs do not predict the exact time of death.
b. Death is inevitable, it will come anytime now.
c. You are saddened that your mother is dying.
d. Are you worried that your mother will die?
4. The client died with her family around her. The children are crying hysterically and hanging on to their
mother. What nursing action is BEST for you to take?
a. Ask physician to prescribe tranquilizer for the family members
b. Allow the family some privacy and time to be with the client before doing after care
c. Allow the family to view the body then transport the body to the hospital morgue immediately
d. Reassure the family that the body of their love one will cared for
5. Having witnessed the death of Mrs. Marina, you become aware of the tears that are welling in your
eyes . It is most appropriate to remember that when caring for a dying client
a. The nurses emotional response sets an example as to how the family should grieve
b. The nurses own feelings and thoughts about death influences her own ability to care for the
client and the family
c. Any show of emotions by the nurse is considered non-therapeutic
d. The nurse should send the family out when bathing the body and placing identification tags
Situation 2-Mr. Rivera , 54 years old, is admitted to the medical unit for executive check up his
admitting notes reveal; temperature: 36.8 degrees C; pulse rate: 86/minute; respiratory rate: 18/minute;
BP: 160/90 mmHg.
6. Mr. Rivera , You most important INITIAL nursing action would be to
a. Take him around the ward to show him the unit set up
b. Introduce Mr. Rivera to the other staff in the unit
c. Obtain Mr. Riveras nursing history
d. Identify needs of Mr. Rivera that may require immediate management
1

7. The Charts admission notes state that Mr. Rivera has bi-pedal edema. During assessment you
VERIFY this by;
a. Doing palpation
d. Checking the results of laboratory
b. Interviewing the client
tests
c. Doing inspection
8. Mr. Rivera asks you what has should do to help reduce swelling of his feet and ankles. Your most
appropriate response would be the following EXCEPT:
a. Elevate his feet while sealed or while lying in bed
b. Reduce intake of salty food
c. Request the doctor for diuretics
d. Inform him that edema is caused by problem with his kidney
9. Mr. Rivera had blood extraction for hematology, blood chemistry, lipid profile, FBS. The laboratory
results are in. Of the following which is not within normal?
a. Cholesterol 4.28 mmol/L
c. FBS:6.5 mmol/L
b. HBA1C: 5.7%
d. Hematocrit; 39.7/L
10. You are planning Mr. Riveras discharge from your unit. Your nursing responsibilities include all
EXCEPT:
a. Making a final assessment of the client
b. Replying to queries regarding his hospital bill
c. Giving instructions regarding his home medications
d. Arranging for his transportation home
11.
12. SITUATIONS 3- Nurses in the medical unit are finding ways to improve collaborative relationship
with the physicians. Efforts to identify factors that foster or impede nurse-physician
collaboration are thoroughly examined. A review of the antecedent environmental factors was
undertaken.
13. A variety of factors collected have been linked to the quality of nurse-physician collaboration. Given this
information, which of the following measures is appropriate to foster nurse-physician collaboration?
a. Present collected data to the physicians and those involved in health care
b. Nurse and physician should be willing to consider each others position
c. Consider both environmental and professional factors
d. Arrange a meeting to discuss issues and concerns with the physician and other members of the
health team.
14. Information gathered by nurses show the importance of nurse-physician communication. With the
current recognition that many medical errors are caused by communication failure, which of the
following most appropriate intervention?
a. Conduct in-service education for nurses to improve competencies to address the issues.
b. Organize a conference on medication error participated by nurses, physicians and other
c. Involve a form o f communication where 2 parties engage in problem solving discussion
d. Develop a policy where all members of the health team can use it as a reference
15. There are other problems that were identified during the gathering of data but at the situation not have ready
solutions. In this situation, which of the following is the most appropriate action to be undertaken?
a. Nurse and physician should identify types of problems amenable to collaboration
b. Give priority to these problems because they are good candidates for collaboration
c. Immediate action must be done to assist the clients in their health problems
d. Prioritize problems needing immediate attention and solution
16. Given the above situation, which of the following actions should be done by both parties?
a. Continuous conference to keep track of the concerns of the clients
b. Conduct weekly conference for continuity of medical management
c. Encourage open communication for effective collaboration
d. Design condition to support desired levels of collaboration more effectively
17. Of the following nursing service staff, who are MOST likely to engage in problem solving communication with
the physician to ensure the quality of communication and outcomes?
a. Nurses who are on the managerial level to ensure effective problem solving with physician
b. A team of experienced nurses with less experienced staff nurses
c. Any staff available to the conference and collaboration
d. Nurses assigned to the clients with identified problems
18.
19. Situation 4: Therapeutic communication forms a connection between the client and the nurse.
Furthermore, it facilitates the establishment of the nurse-client relationship and fulfils the purposes
of nursing:
20. Mrs. Irma Santos, 45 year old, has terminal cancer of the breast. She cries and tells the nurse. Why do I
have to suffer this kind of illness? There is no cure for this and I wish my family would not hope for a cure.
Which of the following is most appropriate response of the nurse?
a. Is your family ready to accept your condition?
b. You feel angry that your family hopes for a cure for your illness?
c. You sound that you are likely to die.
2

d. I think you and your family should discuss your condition with your physician.
21. While on your way to the cafeteria, you were greeted by a friend who happens to be visiting a client under
your care. She asks about the clients condition. Which of the following would be the most appropriate
response of the nurse?
a. I am not in position to discuss her condition but you are my friend. I can tell you that she is on her way
to recovery
b. If you want to know her condition, why dont you talk to her attending physician?
c. I cannot discuss the status of the client with you.
d. Confidentially, I can tell you that her condition is unstable.
22. An 18 years old client has been in the hospital for 3 days with infection of clamydia. While administering her
12:00 noon medication, the client tells the nurse that she has a secret which she wants the nurse to know
but asked the nurse not to tell anybody. Which of the following is MOST appropriate response of the nurse?
a. What you will tell me will be properly documented.
b. Yes, I promise to protect you when in entrusting your secret to me.
c. Yes you can trust me not to divulge your secret.
d. I cannot promise to keep a secret if it affects your health.
23. A depressed client tells the nurse that she is very disappointed following her loss of job. Im a failure and
cannot perform my work right. The following are appropriate responses of the nurse EXCEPT:
a. Provide experiences that will enhance her self-esteem
b. Reassure the client that everything will get better soon
c. Stay with the client and listen to what she says
d. Motivate the client by giving positive support and encouragement
24. The nurse is interacting with a client who verbalized that she is hearing voices telling her that she is a bad
girl. Which of the following responses is the MOST appropriate?
a. I understand what you feel but keep calm.
b. Nobody is around expecting the two of us.
c. Dont worry it will not harm you.
d. Its difficult for you to understand all that you are experiencing right now.
25.
26. Situation 5: Nedy, a now Registered Nurse, applied for work as a research assistant. A thorough
understanding of the steps in the research process is important when doing a study.
27. Nurse Nedy knows that defining the purpose of the research project serves which function?
a. States the focus of the research study
b. Identifies population group to be used
c. Determines statistical treatment needed
d. Explains why the physician is significant to study
28. During her interview, Nurse Nedy was asked which type of research is intended to gain insight by
discovering meaning? Her best reply is:
a. Phenomenological research
c. Quantitative research
b. Qualitative research
d. Anthropology based research
29. When another nurse tells Nedy that she performs hand washing eight times a day but cant explain why
except to say Ive always done it his way her answer is an example of
a. Scientific knowledge
c. Unsubstantiated knowledge
b. Authoritative knowledge
d. Traditional knowledge
30. Nurse Nedy is doing research on the effect of cholesterol on blood pressure. Blood pressure is what type of
variable?
a. Intervening variable
c. Exploratory variable
b. Independent variable
d. Dependent variable
31. The term used to refer to information collected in research is:
a. Abstract
c. Data
b. Mean
d. Subject
e.
f. Situation 6: Nurses have a responsibility to understand the current legal and ethical
guidelines that govern the practice the nursing profession.
32. A new registered nurse is being interview for a staff position in a private hospital. Which of the following
statements indicates her understanding of the practice of nursing in the Philippines as provided for nursing
law?
a. Only professional licensed nurses can practice nursing in the Philippines.
b. A registered nurse can practice professional nursing in the hospital and community setting.
c. A nurse is certified to practice nursing in the Philippines.
d. A registered nurse license provides basis for professional nursing practice.
33. A nurse assigned in the Surgical Unit visited the mother of her friend confined in the Medical Unit. She was
observed by the staff nurse in the unit reading the chart of the client in this situation the action of the nurse is
described as:
a. Acceptable because she is known to the client
b. Unethical because of possible breach of confidentially
c. Appropriate because she is part of the nursing staff of the hospital
d. Inappropriate because she is not assigned in the unit

34. The nurse is concerned about the medical care of her client who has been confined in the hospital for
2 weeks . she has a physician friend not connected with the agency whose opinion she asked
regarding the treatment of care. In this case, the nurse acted:
a. In accordance with hospital policies and regulations
b. In violation of the principle of confidentiality
c. Following the appropriate chain of command
d. Based on what is good for the client
35. A nursing student asks permission from the head nurse to photocopy the record of the client she is
presently taking care of. She is present a case study and needed substantiate her data. Which of the
following should be appropriate action of the head nurse?
a. Tell the nursing student to ask permission to the attending physician
b. Allow her to photocopy the pages related to the information needed
c. Allow her to write down pertinent but no identifying information
d. Do not allow photocopying due to confidentiality
36. A nurse researcher is conducting a research study on the concerns of the elderly regarding
hospitalization. An elderly client was offered to participate in the study. She signed the consent but later
decided to withdraw from the project. In this situation the elderly client
a. May withdraw as long as the family requests withdrawal
b. May withdraw at any time of the study
c. Cannot withdraw since the consent is a legal document and has been signed
d. Cannot withdraw since the study has started
e.
f. Situation 7: Nurse Nita is conducting a clinical assessment of Lola, 40 year old female
client admitted to chronic renal disease.
37. Nurse Nita utilizes the most reliable indicators of Lolas fluid balance status which include the following
EXCEPT:
a. Her daily weight record
c. Complete blood count
b. The measurement of intake and
d. Results of urinalysis
output
38. While assessing Lolas skin, nurse Nita noted that the skin flattens more slowly after the pinch is
released. This is an indication that the client is manifesting sign of
a. 2+ pilling edema
c. Fluid volume deficits
b. Shift of body fluids
d. Fluid overload
39. Nurse Nita continues with the assessment of the neck and instructed the client to lie flat on bed. With
the presenting complaints the nurse expects to note which of the following?
a. Neck warm to touch
c. Jugular venous distention
b. Collapse of neck veins
d. Difficulty in moving the neck
40. During the planning phase nurse Nita prioritizes nursing interventions to support clients treatment of
expected outcome. In this case the goal of care for the client should be
a. Proper fluid balance of intake and output is attained
b. IV site should be free from infection
c. Prevent depletion of fluids
d. Relief from vomiting and diarrhea
41. The following nursing interventions to prevent or correct fluid, electrolyte , and acid base imbalances
include the following EXCEPT
a. Fluid and blood products replacement
d. Appropriate patient and family
b. Allaying of anxiety
teaching
c. Modification of fluid intake
e.
f. Situation 8: Nursing practice act requires nurses to maintain a safe environment for their
clients. Nurses must act to identify and minimize risks to clients.
42. A nurse is taking care of Mr. Antonio Cruz who is receiving oxygen therapy. A watcher approached her
saying there is fire burning in the trash basket inside the medication room in the nurses station. What
INITIAL action should nurse do?
a. Turn off the oxygen and remove all clients from the room
b. Get the fire extinguisher to put off the fire
c. Calm the clients and escort them to a safe area
d. Ask for help from the visitors
43. What nursing action is essential when Mr. Cruz is to have oxygen administration at home?
a.
b.
c.
d.

Assist the client and family check all electrical appliances in the vicinity for extension cords
Turn off all electrical devices inside the room of the client
Instruct the clients to install a carpet inside the room
Instruct relatives to have fire extinguisher ready
e.
f.
4

44. Anna, a change nurse in the pediatric unit, is assessing the area for fire hazards. The following situation is
considered the GREATEST fire hazard:
a. Cleaning supplies and cardboard boxes stored in the room with oxygen tank
b. Closet of clients filled with clothing and newspapers
c. Personal items of clients kept under the bed
d. Some staff smoking in the rest room
45. While doing her rounds, the nurse passed through a private room and saw flames and smelled smoke.
Which of the following should be INITIAL action of the nurse?
a. Evacuate all clients in the building
c. Evacuate the clients out the burning
b. Ask for assistance
room
d. Evacuate the weakest clients first
46. The nurse explaining universal precaution to the client. The primary purpose of universal precaution as part
of maintaining safe environment is to:
a. Prevent health workers from acquiring communicable diseases
b. Reduce the spread of the disease
c. Prevent nosocomial infection
d. Prevent the spread of communicable diseases
47.
48. Situation 9: It is rainy season and the pediatric clinic where you are assigned is filled with children
and mothers waiting for attention and treatment.
49. Many children in the clinic have upper Respiratory Tract Infection (URTI). Alice has two children with her at
the clinic. To prevent the spread of URTI, the BEST instruction to give mothers like ALICE will be to:
a. Teach child to use sleeves to wipe off nasal discharges
b. Instruct mother and child to wear protective masks at all times
c. Wipe off childs nasal discharge so that no mucous crust forms on the nostrils
d. Wash hands thoroughly with soap and water after handling mucous discharges
50. The nurse teaches Alice and other parents that URTI spreads through droplets after coughing and sneezing.
Your health instructions are effective when the parents do the following EXCEPT:
a. Deposit sputum in tissue and discard used tissue in a trash can
b. Cover mouth and nose when coughing or sneezing
c. Wash and dry hands by using a towel provided in the lavatory
d. Wash hands thoroughly after contact with mucus secretions
51. Following the nurses instructions on how to prevent spread of infection. Alice teaches her children how to
prevent infecting their playmates when they have URTI. Which of the following actions would be considered
INEFFECTIVE in preventing spread of infection?
a. Washing hands after blowing nasal discharges
b. Covering mouth and nose when sneezing or coughing with their skirt or shirt
c. Covering nose or mouth with hands when sneezing then continue playing
d. Pinning a handkerchief or face towel to wipe off mucus secretions or cover nose
52. You demonstrate proper hand washing technique to the parents in the clinic as a step to prevent spread of
infection. The parents perform the practical procedure correctly when they.
a. Rub hands together in between fingers , using soap and rinse with running water
b. Rub hands together for friction under running water
c. Wash fingers with soap and rinse with water in a basin
d. Wash hands with antimicrobial soap, apply rubbing alcohol , dry hands by allowing alcohol to
evaporate
53. At home, Alice observes principle of infection control when she:
a. Avoids shaking linen, clothes and towels used by a sick child
b. Keeps kitchen untensils and paltes in cupboards where leftover food are stored
c. Places handbags and baskets on food preparation areas
d. Avoids shaking and stores used clothes and linen in the clothes cabinet
54.
55. Situation 10:A number of clients in your unit are at risk of developing pressure sores. As a
precaution, the supervisor emphasizes the nurses responsibility in ensuring proper care of clients
with problem of immobility.
56. While changing linen of Aling Mila, a comatose client. The nursing aide reports that she noticed a reddened
area in the left buttock of the client. Upon inspection, you noted that the area blanches and is the size of a
peso coin. Your MOST appropriate immediate nursing approach would be to:
a. Measure the size of the reddened area for proper documentation
b. Instruct the nursing aide to finish bed making using dry fresh linen
c. Endorse a schedule for turning and positioning the client round the clock
d. Position the client of her right side
57. To decrease the occurrence of pressure sores on Aling Mila, the nursing teams goal is to reduce pressure
points. The MOST appropriate nursing intervention would be to
a. Elevate the head part of the bed as little as possible
b. Massage over the bony prominences
c. Use a donut cushion while client is seated
d. Place the client on a side lying position

58. While assessing clients assigned to your care, you observe that the client with the greatest risk for
developing a bedsore or pressure sore would be:
a. 4 year old girl in Bucks traction
c. 82 year old client who has had mild
b. 40 year old unconscious client
stroke
d. 70 year old client type 2 diabetes
59. While assessing the pressure sore of a 75 year old client, the nurse documents that healing is taking place
when she observe as presence of
a. Eschar
c. Granulation tissue
b. Exudates
d. Ragged edges around the wound
60. In a nurses meeting in the ward, the senior nurse discusses prevention of pressure sores. She identifies
practices that are most likely to cause shearing injury to the skin and should therefore be avoided. Of the
following practices, which one causes the LEAST harm to the client?
a. Failure to use a draw/lift sheet when moving client to the head of the bed
b. Failure to lower the head part of the bed before moving the client upward
c. Positioning an immobilized client without help from staff
d. Dragging the heels of the client in bed as he is being positioned
61.
62. Situation 11: Primary prevention involves health promotion as protection against disease. Activities
of this type generally apply to the healthy individuals before any disease or dysfunction occurs.
63. Nurses play a big role in the primary level of prevention. Examples of nurse activities showing primary
prevention are the following EXCEPT;
a. Referrals to client support groups like those for cancer patients
b. Teaching parents of toddlers about prevention of poisoning and accidents at home
c. Family planning classes to newly weds
d. Giving immunizations to children
64. Secondary prevention includes health maintenance activities which involves the following EXCEPT:
a. Nursing care to maintain skin integrity of a diabetic client
b. Giving medications and treatments to discharged clients
c. Proper positioning of clients with disability in the home setting
d. Smoking cessation program
65. When teaching your clients about nutrition, you include the following food as rich sources of good
cholesterol, EXCEPT:
a. Fish
c. Soya
b. Beef
d. Olive oil
66. A community based hospital offers acute care in addition to adult outpatient services exercise and yoga
classes for young and old. This hospital provides which type of services?
a. Tertiary and illness prevention
c. Secondary and tertiary
b. Primary and tertiary
d. Primary and secondary
67. Mr. Tan 48 years old is attending a smoking cessation program to be held at the mearby high school
conducted by the school nurse. This program is classified as
a. Diagnosis and treatment
c. Rehabilitation
b. Health restoration
d. Health promotion
e.
f. Situation 12: To carry out management functions in any health care setting it is necessary for
the nurse to integrate leadership skills that he/she developed.
68. To improve quality client care the nurses created problem solving committeesheaded by senior nursing
staff to review standards of care and develop policies and procedures. Its desired result is best seen in;
a. Continuous evaluation of nursing practice/protocols in relation to desired patient outcomes
b. Allowing changes in staff rotation plan to accommodate personal needs of the staff
c. Increasing staff communication like providing a bulletin board for sharing information among
personnel
d. More nurses participating in doctors rounds and giving immediate information to doctors regarding
patient status
69. The nursing departments organizational chart illustrates structure and relationships of the nursing leaders
and staff of the organization. The following are the functions of an organizational EXCEPT:
a. Lists functions and duties of the staff
b. Illustrates centrality of control in the organization and chain of command
c. Indicates relationship of leaders to other management staff
d. Identifies managerial levels
70. A hospital is constructing a new wing and the Director of Nursing is asked to help design it. To achieve
maximum efficiency in carrying out nursing activities the Director of Nursing would consider which of the
following conditions to be most helpful?
a. Environmental factors such as current economic, legal, technological and social influences that the
organization must consider
b. How to structural plan facilities staff interaction and the rituals the nurses use to conduct work
c. Work flow where equipment , medication and other items essential for patient care are stored and
positioned
d. Type of equipment and technology and its effects on how work tasks are designed and carried out.

71. The management is important to provide quality care and prioritize work. The nurse finds the following
practices helpful in managing time for patient care except:
a. Keeping telephone communication short
b. Blocking out time to accomplish important activities
c. Doing time and motion study to determine time utilization
d. Dealing with interruption openly and directly
72. The Director of Nursing wants to improve the quality of health care in the hospital. The following activities
are examples of quality assurance measurements EXCEPT:
a. Evaluating outcomes or end results care provided to client
b. Asking clients to accomplish client satisfaction survey forms
c. Measuring quality of care against established standards of nursing care
d. Checking if emergency carts or medications are properly stock
e.
f. Situation 13: Mr. Reden, 52 years old, known diabetic (TYPE 2) is admitted with symptoms of
high blood pressure of 190/100mmHg, an unhealed wound on his right big toe and has bipedal edema. He is on insulin. Co-management by health team is recommended in the care of
Mr. Reden.
73. Based on the presenting condition of the client upon admission, the nurse would immediately refer Mr.
Reden to a/an;
a. Endocrinologist for management of diabetes mellitus
b. Dietitian for nutritional management
c. Cardiologist to stabilize blood pressure
d. Diabetes nurse educator for self management of symptoms
74. The head nurse calls for a meeting of the staff nurses to plan care management to Mr. Reden. The priority
nursing action in the care of the client would be:
a. Blood glucose monitoring
c. Accurate measurement of fluid intake
b. Monitoring of blood pressure
and output
d. Accurate insulin administration
75. Upon assessment of the client lower extremities the nurses notes the unhealed condition of the clients
infected toe wound. The nurse would
a. Prepare equipment to wash and disinfect the affected toe
b. Refer to attending physician for proper wound management
c. Call the head nurse assistance as the nurse starts wound debridement
d. Call relative to gather information about cause of wound infection
76. The nurse learns the client does not regularly do blood glucose monitoring and still has not leaned how to do
self-administration in insulin. To learn these you will refer the client to:
a. Pharmacist
c. Diabetes nurse educator
b. Advance practice nurse
d. Medical intern
77. Mr. Redens condition improves and discharge planning is initiated. This includes planning his nutritional
regimen to encourage compliance. The client MOST likely be referred to
a. Nutritionist
c. Dietitan
b. Endocrinologist
d. Cardiologist
e.
f. Situation 14: A nurse is assigned to several clients and her functions include giving
intravenous (IV) medications and fluids. During the end of shift endorsement she receives
incoming doctors orders to run some IV fluids for clients assigned to her.
78. Mr. Roman, 49 years old, has a doctors order to receive 1 liter of normal Saline solution to run for 24 hours.
The nurse would set the intravenous fluid to infuse at how many milliliters (ml) per hour and how many drops
per minute if the drop rate of the IV tubing is 15 drops/ml?
a. 42 ml/hours, 10 drops /minute
c. 50 ml/hours, 18 drops /minute
b. 48 ml/hours, 15 drops /minute
d. 36 ml/hours,7 drops /minute
79. Bayani, 8 years old, has an order for D5 Lactated Ringers 250 ml to infuse for 4 hours, starting at 8AM,
using IV tubing set with a drop factor of 60 microdrops (gtts)/ml. What should be the rate of flow if the IV is to
be consumed at 12 noon?
a. 48 gtts/minute
c. 43 gtts/minute
b. 63 gtts/minute
d. 58 gtts/minute
80. While reading the doctors orders for the other client you will seek clarification from the doctor for which of
the following orders?
a. Infuse .9% normal saline to keep vein open (KVO)
b. Incorporate 20 mEq potassium chloride in 1 liter of D5 water at 50 ml/hour
c. Flush peripherally inserted central catheter (PICC) with 10ml normal saline every 6 hourse
d. Infuse 500 ml of normal saline for 2 hours
81. Mr. Roldan is a newly admitted to the ward and before administering IV medications you read in his chart
that he has a peripherally inserted central catheter (PICC) that is now 4 weeks old. Upon examination, you
observed that the site is clean and free from manifestations of infiltration, irritation, and infection. Your most
APPROPRIATE action would be to:
a. Document observation in the nurses notes to inform the physician and other nurses
b. Discontinue the PICC lina since it is 4 weeks old
c. Administer the medication as ordered

d. Give medications through oral or intramuscular route


82. While assessing Mr. Pintors IV site you swelling and tenderness above the site. Your most
APPROPRIATE nursing action would be
a. Apply cold compresses to the IV site
b. Stop infusing IV fluids
c. Flush the catheter with normal saline solution
d. Elevate extremity to facilitate drainage by gravity
83.
84. Situation 15: While in the ward, you assigned to clients with problems related to the
gastrointestinal tract:
85. The nurse is preparing Mr. Lim for cleansing enema. When administering enema the maximum height
at which the enema can should be held from the level of the bed is
a. 14 inches
c. 16 inches
b. 10 inches
d. 12 inches
86. While administering the enema, Mr. Lim complains of abdominal cramps. Which of the following would
be the MOST appropriate action of the nurse?
a. Clamp the tubing few minutes till the cramps subside, then continue
b. Pull the rectal tube slowly till the cramps subside
c. Stop the procedure and refer to the attending physician
d. Lower the enemia can to slow down the inflow of the enema solution
87. Following surgery Mrs. Mora developed abdominal distension. The physician ordered a rectal tube
insertion, to relieve distension to achieve maximum effectiveness, how long should the rectal tube be
left in place?
a. 5 minutes
c. 30 minutes
b. 15 minutes
d. 40 minutes
88. For ensuring that the nasogastric tube (NGT) is in place, the nurse prepares to feed Mrs. Mora using
the open system. With a 30 ml syringe, the nurse proceeds with feeding following this sequence.
89. 1. Hold the NGT high to prevent backflow and then clamp
90. 2. Open clamp and raise or lower the syringe to regulate flow of formula
91. 3. Remove the plunger of the syringe and attach the NGT
92. 4. Add 30 to 60 ml of water to irrigate syringe allowing it to run down the NGT
a. 2,3,4,1,5
c. 3,4,2,5,1
b. 1,3,2,5,4
d. 4,3,1,2,5
93. The nurse is to perform gastric gavage. What should be the best position of the client while the gastric
tube is being inserted?
a. Supine position
c. Trendelenburg position
b. High : fowlers position
d. Low fowlers position
e.
f. Situation 16: Proper Nutrition and elimination are important to health and the nurse has an
important role to play in assessing people from various age groups obtain proper take.
g.
94. Roman, 36 years old , is diagnosed with peptic ulcer and asks you what food is best to add to his diet
so as not to exacerbate his symptoms. Your BEST response would be for him to take
a. Leafy green vegetables dishes
c. Mocha, cafe latte and other similar
b. Citrus fruit juices or shakes
drinks
d. Milk regularly 3-4 times daily
95. You are assigned to Mrs. Dulay, a client with an order for cleansing enema. While doing the procedure,
the client groans and complains of abdominal cramping, your MOST appropriate initial nursing action
would be to:
a. Reduce the flow of the fluid by clamping the enema tubing
b. Instruct the client to relax , inhale slowly
c. Lower the height of the enema container
d. Push the rectal tube further in by 2 inches
96. An elderly client you are taking care of has fecal incontinence for 3 days now. He is able to tolerate food
but has no control of his bowel movement. He has soft watery stools and uses adult diapers. While
caring for this client you will watch out closely for risk of:
a. Increased abdominal cramping
b. Perineal and anal skin breakdown
c. Malnutrition and weight loss
d. Falls when he tries to go the bathroom
97. Dennis, 5 years old, is brought to the hospital for severe diarrhea. You are aware that a major problem
that may develop that will adversely affect Dennis would be
a. Severe abdominal cramping
b. Excessive passing of flatus
c. Severe fluid electrolyte imbalance
d. Irritation of the anal sphincter

98. Miss Reyes a client who had abdominal surgery under general anesthesia , is still in the recovery room, you
are aware that clients who went through general anesthesia will most likely experience:
a. Absence of peristalsis
b. Tolerance for soft diet immediately after operation
c. Immediate return of gastrointestinal motility
d. Excessive gas formation noted upon auscultation
99.
100.
Situation 17: Mariza , a staff nurse in the surgical ward, has been assigned to take care of Mrs.
Jose , a 58 year old client who has an endotracheal tube
101.
Nurse Marizas objective is to improve clients respiration after she noted thickened tenacious
secretions. To loosen the secretions, the MOST appropriate nursing intervention is to:
a. Instill mucomyst into the endotracheal tube and frequently turn client unless contraindicated
b. Administer humidified oxygen and place in side lying or prone position unless contraindicated
c. Increase fluid intake and ask client to do deep breathing and coughing exercises
d. Assess clients respiratory status and perform clapping to looses secretions
102.
Nurse Mariza performs endotracheal suctioning. The nurse appropriately does the suctioning
procedure when she performs which of the following:
a. Rotates the catheter gently and suctions for not more than 10 seconds each time
b. Observes and records the amount and character of the secretions after each suctioning
c. Assesses the respiratory and circulatory status after a cluster of 5-8 times suctioning
d. Observes how long the client tolerates the catheter during the suctioning process
103.
In the care of this client the nurse monitors the cuff pressure and takes care to reduce the risk of
treatment tissue necrosis by maintaining the cuff pressure to:
a. 30-35 mm Hg
c. 40-45 mm Hg
b. 10-15 mm Hg
d. 20-25 mm Hg
104.
When taking care of Mrs. Jose, Mariza performs oral and nasal care every 2 to 4 hours to promote
hygiene and comfort. As a precautionary measure for possible biting down of the oral endotracheal tube.
The nurse should
a. Request an assistant to hold the patient down
b. Use an orophryngeal airway
c. Provide humidified air prior to the procedure
d. Place the client on side lying position
105.
The head nurse reminds Nurse Mariza about measures that must be strictly observed when
suctioning the client through the endotracheal tube. This measure is
a. Turning on the suctioning apparatus during catheter insertion
b. Suction by rotating 2 to 3 times before withdrawing the catheter
c. Always use rubber gloves when suctioning to prevent infection
d. Hyperoxygenating the client before and after the procedure
106.
107.
Situation 18: Clients record is a structured device where all tasks concerning the diagnostic
and treatment process done on the client are documented. An account of what has occurred
between the client or the health care team has to be recorded once interaction has been undertaken.
108.
An entry in the nurses notes for a client with urinary tract infection states. Encouraged fluid intake to
2,500 ml per day. Which description of the nurses statements applies?
a. It describes the amount of fluid intake desired
b. It established accuracy using an exact measurement
c. It is incorrect as it lacks accuracy of measurement
d. It does not specify fluids allowed
109.
The nurse is recording the treatments administered to her clients. The following information should be
included in her charting , EXCEPT:
a. Health teaching
c. Time administered
b. Clients response compared to previous
d. Equipment used
treatment
110.
A male nurse is giving a change of shift report for all clients in the medical unit at the nurses station.
During this reporting the nurse is expected to:
a. Review the condition of the client by reading the documented information
b. Report condition of the client and compare with the incoming staff need to know
c. Provide significant information about the client as baseline for the next shift
d. Read the data about the client objectively
111.
The nurse is preparing Mr. Nick Gomez for transfer from the Intensive care Unit to his private room.
To promote continuity of care what information should be included in the transfer report?
112.
1. Clients name, age, physical and medical diagnosis and allergies
113.
2. Correct health status of the clients at the time of transfer
114.
3. Any critical observation to help the receiving nurse
115.
4. Need for special equipment
a. 1 and 2
b. 3 and 4
c. 1,2,3 and 4
d. 1,2, and 4

116.
Mr. Douglas Nava, a 55 year old executive, requests the nurse if he can read his medical records
upon discharge. What is the most appropriate action of the nurse?
a. Allow the client to read his chart because of his clients right
b. Tell the client that he is not allowed to read his chart
c. Ask the client to write a written request
d. Refer the request of the client to the physician
117.
118.
Situation 19: Nurse Carmen is coping with transition from student nurse to a professional
nurse. Along with an accumulation of knowledge, skills and competencies she is leaving
enough space for her unique personality to develop. Using Benners Stages from Novice to
Expert the following questions apply.
119.
120.
Which of the following stages begins in nursing school?
a. Expert
c. Proficient
b. Novice
d. Advanced beginner
121.
As a new graduate nurse Carmen begins nursing practice as a/an:
a. Novice
c. Competent
b. Proficient
d. Advanced beginner
122.
Nurse Carmens performance as a new graduate is characterized as:
a. Having feeling of mastery
b. Possessing intuitive grasps
c. Formulating principles
d. Exhibiting rule governed behavior
123.
Nurse Carmen successfully passed the Nurse Licensure Examination. She is now employed as
staff nurse in a General Hospital. How long will it take approximately for nurse Carmen to achieve the
competent level?
a. 12 months
c. 6 months
b. 18 months
d. 30 months
124.
In order to attain the expert level nurse Carmens experience level should be:
a. Innovative
c. Extensive
b. State of the art
d. Varied
e.
f. Situation 20: Nurses are obliged to fulfil their responsibility and provide ethical and moral
care that demonstrates respect for other.
g.
125.
The nurse manager is preparing staff development classes for new nurse. Which of the following
should be included in relation to ethical decision making?
a. Ethical decisions arrived at for client care are based on the recommendation of family and
significant other
b. Ethical decision making is based on knowledge , facts, and strong commitment to right or wrong
c. Ethical decision making is the responsibility to the nurse alone
d. Ethical decision making is based on the philosophy of individual values and beliefs
126.
Ria, daughter of the client, refuses to inform her father about his diagnosis. The nurse is
concerned about whether or not she will tell the client about his diagnosis. This is an example of an
ethical:
a. Conflict
c. Dilemma
b. Concern
d. Issue
127.
After the client was informed by the physician that he is positive for stage IV cancer of the
prostate. He requested the nurse to withhold the information from his wife and children. Which of the
following is an appropriate action of the nurse?
a. Encourage the client to tell his wife
b. Refuse to do the request but offer support and guidance
c. Pretend not to have understood the request and consult supervisor
d. Ask patient to give her time to think about it and refer to the physician
128.
When the nurse finished performing foot care on the client , she was requested to come back to
change the linen. The nurse changed the linen as requested by the client. The nurse is demonstrating
which of the following ethical rules?
a. Non maleficence
c. Justice
b. Confidentiality
d. Fidelity
129. Morning shift is over and the outgoing nurse was about to leave the unit when the relative of the client
called because the client fell out of bed. The nurse hurriedly went to the clients room to attend to the
client. Which of the following ethical principles illustrate the action of the nurse?
a. Justice
b. Autonomy
c. Beneficence
d. Nonmaleficence
10

130.
131.

132. A1 PASSERS TRAINING, RESEARCH, REVIEW AND DEVELOPMENT COMPANY


133.
FINAL COMPREHENSIVE EXAMINATIONS
134.
135.
NURSING PRACTICE II-Community Health Nursing and Care of the Mother and
Child
136.
137.
138.
GENERAL INSTRUCTIONS:
139.
1. This test booklet contains 100 test questions.
140.
2. Read INSTRUCTIONS TO EXAMINEES printed on your answer sheet.
141.
3. Shade only one (1) box for each question on your answer sheets. Two or more boxes shaded will
invalidate your answer.
142.
4. AVOID ERASURES.
143.
144.
INSTRUCTIONS:
145. 1. Detach one (1) answer sheet from the bottom of your Examinee ID/Answer Sheet Set.
146. 2. Write the subject title Nursing Practice I on the box provided.
147. 3. Shade Set Box A on your answer sheet if your test booklet is Set A; Set Box B if your test booklet is
Set B.
148.
_________________________________________________________________________________
_________
149.
150.
SITUATIONAL
151.
152.
Situation 1- Nurse Dang Casillo, initiated the organizing of the Caring Frontiers Nursing
Service Companyin Leyte. This was envisioned to deliver health care and various nursing services
through home health care services well within the scope of nursing practice.
1. Nurse Lanie is one of their Registry Officials and acts as a liason between families and communities served
by the company to actively engage on policy and social change that will support and promote family health
in their Province. Nurse Dang Castillo is performing what role?
a. Care provider
c. Client Advocate
b. Counsellor
d. Facilitator
2. Formulating and implementing a supervisory plan, monitoring and evaluating beginning nurse practitioners
performance in the implementation of public health programs are what functions of a public health nurse?
a. Trainer
c. Supervisor
b. Coordinator
d. Manager
3. As an advocate, Registry Nurse Lanie places her clients rights as priority. She is aware that advocacy work
involves which of the following?
a. Influencing public opinion
b. Obtaining a general information about the community
c. Coordination with the health team
d. Prioritizing health conditions and problems
4. According to the World Health Organization (W. H. O) one of the leading cause of mortality in the Philippines
is which of the following?
a. Leukemia
c. Malignant neoplasm
b. Heart disease
d. Lower respiratory tract infections
5. Registry Nurse Lanie is conducting a community diagnosis composed of Demographic variables, socioeconomic variables, health and illness patterns, and health resources and political/leadership patterns. What
type of community diagnosis is nurse Lanie conducting?
a. Individualized Nursing Diagnosis
b. Population Focused Diagnosis
c. Comprehensive Community Diagnosis
d. Problem Oriented Community Diagnosis
6.
7. Situation 2: A nurse assigned to the community health center where the variety of experiences and
the culture of the community influence the care she gives to the clients.
8. While in the community health center , you note various practices among the families you serve. Which of
the following situations will be OF CONCERN to you as the nurse?
a. Mang Tomas wears a copper bracelet for his rheumatoid arthritis
b. Mang Andoy gives his son various herbs everyday
c. Aling Pat applies Vicks VapoRub to the chest and back of her grandson with colds
d. Aling Mika uses Atis seeds on the hair of her granddaughter to treat lice
9. Mr. Cruz, diagnosed with Alzheimers Disease, is brought to the center by his daughter to seek help
regarding home care. In planning the care of Mr. Cruz priority should focus on
a. Providing food rich in fiber to prevent constipation
b. Protecting the client from possible injury in his environment
11

c. Assisting the client to perform activities of daily living


d. Assisting all family members deal with the challenges of long term care of this client
10.

11.
12.
13.
14.
15. You are following up a client who has cataract. The most important nursing action/instruction to
implement would be to:
a. Advise the client to wear dark glasses indoors and outdoors to guard v.s sun glare
b. Provide adequate lighting at home at all times
c. Instruct family not to change furniture arrangement at home
d. Advise the client to have the cataracts remove
16. While on duty at the community health center, some clients are brought for consultation. Based on your
assessment, the FIRST client to attend to would be
a. Aling Juana, with Demetia, who wanders in the streets every morning
b. Mang Rodolfo, 65 year old with congestive heart failure and 3+ pitting edema
c. Aling Maria, 56 year old, terminal client with weight loss (15lbd) last month
d. Mang Joelibert, 70 year old, with Parkinsons disease and started hallucinating
17. You are visiting a newly discharged cerebrovascular accident (CVA) client currently confined to a
wheelchair for long periods of time. You most appropriate PRIORITY intervention would be:
a. Ask the client to move his buttocks every two hours to increase blood circulation to the area
b. Prevent skin breakdown by putting a pressure relieving cushion in the seat of the wheelchair
c. Refer to physical therapist to teach client to transfer from bed to wheelchair
d. Instruct family to feed high protein diet for better skin integrity
18.
19. Situation 3: Documentation is an important aspect of every nurses activity. This is a major area
of responsibility which helps facilitate continuity of work within a 24 hour cycle.
20.
21. A 26 year old mother was admitted for hyperemesis gravidarum. While taking the history of this client it
would be MOST important to report which of the following?
a. The client has cool lower extremity bilaterally
b. The client has diminished palpable peripheral pulses
c. The client is anxious about the effect of her condition to the baby
d. The client has allergy to shellfish
22. You are on duty and you received report from the previous shift. Which of the following clients should
you attend to FIRST?
a. A client who is receiving ciprofloxacin and complains of a fine macular rash
b. A client who is receiving a blood transfusion and complains of a dry mouth
c. A client who is scheduled to receive heparin and the PTT is 70 seconds
d. A client who is receiving I. V. Potassium and complains of burning at the IV site
23. You are reviewing the nurses notes in your clients chart. You would be MOST concerned by which of
the following entries?
a. Foley catheter draining clear urine and pH is 6.5
b. The client drink 3 glasses of orange juice every day
c. The client skin is balanced over the scapular areas
d. Vital signs are within normal limits
24. You are attending to clients in your clinic. As you return to your desk you find 4 phone messages.
Which of the following messages should you return first?
a. A client is nauseated and has vomited 6 times in the previous 24 hours
b. A client , with stage II decubitus ulcer at home reports that the dressing has come off
c. A client is complaining of leg pain after walking haft a mile
d. A client with cold symptoms has an oral temperature of 39.50 C
25. At approximately 6PM, a nurse deployed for duty in one of the affiliate hospitals, begin to open the
nurses notes for the evening shift. The last entry is noted for 1pm and there is no signature, the MOST
appropriate nursing response is to
a. Begin charting on the next line below the last entry inform the day nurse to make a late entry to
complete the chart
b. Do not enter anything until the day nurse has been notified of the problem and returns to the unit
to complete her charting
c. Review with the client the activities after 1pm, and enter what are determined to be the activities
after 1 pm
d. Leave approximately 3 or 4 lines for the day nurse to enter some of her missed entries and sign
the chart
12

26.
27. Situation 4: Myths and faliacies as pertain care of the birthing mothers and their newborn have
been debunked resulting in the development of what is now known as Essential intrapartum
newborn Care or EINC. The following applies:
28. On December 7, 2009, the Department of Health issued Administrative Order implementing the ENC
with the goal of rapidly reducing the number of newborn deaths in the Philippines. With international
standards integrated in intrapartal care the program has now evolved into the EINC program under the
guidance of the W. H. O now considered as myths and fallacies include
29. 1. Use of enema to reduce the risk infections and shorten the duration of labor
30. 2. Shaving the pubic hair of women in labor as a hygienic practice to minimize infection
31. 3. Restricted intake of food and fluid during active labor for possible risk of aspirating gastric contents
due to anesthesia
32. 4. Application of I. V. Therapy to hydrate women due to food and drink restrictions
33. 5. Use of fundal pressure to help the mother in the expulsion of her fetus
34. 6. Early amniotomy and oxytocin augmentation in order to prevent operative delivery
a. 4 , 5 and 6
c. 1,2, and 3
b. None of these
d. All of these 6
35. In the immediate care of the newborn there are also practices which were debunked as practices based
on false. Which are they?
36. 1. Routine suctioning believed to be necessary to clear the babys airway and stimulate him to breathe
37. 2. Foot printing a means of identification for the newborn
38. 3. Early bathing and washing as a form of hygienic practice
39. 4. Routine Separation (baby in the nursery while mother is in her room)
40. 5. Continuance to providing artificial feeding (starts with pre-lacteals then artificial milk substitutes).
a. 2 and 5
c. Only 2
b. 1,3 and 4
d. All 5 are taken now as wrong belief
41. INTRAPARTAL CARE, one among 5 recommended practices include MATERNAL POSITIONING
especially during the 1st stage of labor. The recommended practice now is
a. Allow women to assume an upright position vs. former recumbent (supine/semi- recumbent and
lateral)
b. Strictly follow the traditional medical model of labor and delivery
c. Encourage women to take up the position they find most comfortable to them
d. Allow women to assume any other upright position (walking, standing, sitting, kneeling) vs.
recumbent
42. On ESSENTIAL CARE OF THE NEWBORN, the three (3) new major RECOMMENDED PRACTICES
are:
43. 1. Practice rooming in
44. 2. Skin to skin contact
45. 3. Strict Hand washing of Carers handling the newborn child
46. 4. Use partograph
47. 5. Properly timed cord clamping
48. 6. Initiation of Breastfeeding
a. 1 , 3 and 4
c. 2, 5 and 6
b. 4, 6 and 2
d. 4, 5 and 1
49. Essential intrapartum and Newborn care (EINC) is our countrys instrument in health addressing the
challenge of the United Nations 2 out of 8 Millennium Development Goals (MDGs) targeted to achieved
by 2015. Which these 2 MDGs?
a. MDGs 3 and 6
c. MDGs 1 and 2
b. MDGs 4 and 5
d. MDGs 7 and 8
e.
f. Situation 5: You are passed and registered nurse applying for beginning nursing job.
While waiting you hear you Parish Priest calling for volunteers for a Parish Based Health
Program. You signed up to help and practice your profession. Among health condition
you would normally encounter are obstetrical cases.
50. A 22 year old mother missed 2 of her regular menstrual periods. The Parish Medical Volunteer confirms
an early, intrauterine pregnancy. This is her 1st pregnancy. To determine her expected due date, which
of the following assessments is most important?
a. Dates of last normal menstrual period
b. Date of last intercourse
c. Age of menarche
d. Dates of her 1st menstrual period
51. The action of hormones during pregnancy affects the body by:
a. Blocking the release of insulin from her pancreas
b. Raising resistance to insulin utilization
c. Enhancing the conversion of food glucose
d. Preventing the liver from metabolizing glycogen
13

e.
f.
52. A 28 year old mother has had diabetes mellitus since she was an adolescent. Now she is 8 weeks
pregnant. Hyperglycemia during the first trimester will have what effect on the fetus?
a. Abnormal positioning of the fetus
d. Potential malformation of the fetal
b. Hyporinsulinemia
organs
c. Excessive fetal size
53. You also attend to another young diabetic parishioner-mother who is in her 1st trimester of pregnancy.
As her pregnancy continues what changes in her medication needs should you also anticipate?
a. A steady increase in insulin requirements
b. The variable pattern of insulin absorption throughout the pregnancy requires constant close
adjustment
c. Oral hypoglycaemic drugs will be given several times daily
d. A decrease in the need for short acting insulin
54. A glycosylated haemoglobin level is ordered for a pregnant diabetic because it
a. Will predict how well the pancreas can respond to the stress of pregnancy
b. Gives diagnostic information related to the peripheral effects of diabetes
c. Is the most accurate method of determining present insulin levels
d. Indicates mean glucose level over a 1-to 3-month period
55.
56. Situation 6: Gerele, 32 years old, G2P1, 28 weeks pregnant, visits your clinic and told you that
she has been having various discomforts during this pregnancy and wanted some advises.
57. Gerele had leg cramps that come and go and are extremely painful. The most effective measure that
you can suggests to relieve cramps is to:
a. Lie down and elevate affected leg with a pillow until the cramps stop
b. Extend affected leg with knee straight then bend foot towards the body
c. Increase intake of high phosphorous foods
d. Stand with feet flat on floor and tiptoe alternately, until cramping stops
58. Gerele complains that she doesnt get enough rest and sleep because of her frequent trips to the
bathroom to urinate. The best advise you can give is for her to
a. Bring urine specimen to the bedroom to check for possible infection
b. Bring a commode to the bedroom to reduce trips to the bathroom
c. Hold urge to urinate to improve muscle tone and bladder capacity
d. Drink more fluids at daytime and decrease intake at night
59. Gerele noted that she has beginning varicosities. You informed Susan that varicosities are caused by
pooling of blood in the large veins of the legs. Prevention of this condition involves the following
EXCEPT:
a. Walk around to stimulate blood flow if working in prolonged sitting position
b. When sitting for a long time elevate legs six inches from the floor
c. Do not wear light clothings or crossing legs at the knees
d. When getting out of bed use support stockings above varicosities
60. Gerele complained about difficulty in elimination and worried that she might develop haemorrhoids. You
advise Gerele to
61. 1. Take foods e.g. fresh fruits and vegetables, fruit juices, salads, oatmeal
62. 2. Drink up to 8 glasses of water daily including coffee, tea and soft drinks
63. 3. Exercise regularly and establish a regular time for elimination
64. 4. During elimination use footrest to avoid straining
a. 1,2,3 and 4
c. 1,2 and 3
b. 1,3 and 4
d. 2,3 and 4
65. Gerele tells you that she sometimes do not feel her babys movements, you can instruct Gerele to do
fatal movement count (FMC) or kick counts. These elements are true regarding the fetal movement
count.
66. 1. Client assumes side lying or reclining position and palpates fetal movements for an hour couting for
10 movements
67. 2. Less than 10 movements within two hours may need further evaluation and should be reported to the
nurse
68. 3. Counting fetal movement is best done at the same time daily, when the mother is ready to go to
sleep
69. 4. A well oxygenated fetus moves frequently while a fetus with compromised oxygen supply conserves
energy with less activity
a. 1,2,3
b. 2,3,4
c. 1,2,4
d. 1,3,4
70.
71.
14

72.
73. Situation 7: A current initiative of Department of Health (DOH) is the program called Essential
Newborn Care or ENC. This outlines simple yet meaningful measures to be undertaken by healthcare
worker in doing immediate newborn care management. The following situations apply
74. Nurse Joanna is a member of the birthing team the day Mrs. Maria Luwalhati gave birth to her 1 st born.
Inside the delivery room nurse Joanna assisted the attending obstetrician to address the concerns of
keeping the baby warm, her first step in obtaining thermal protection for the newborn was
a. Drying the baby thoroughly immediately after birth
b. Covering the baby with a clean, dry cloth after the cord has been cut
c. Covering the baby with a clean, dry cloth immediately after birth
d. Drying the baby thoroughly after the cord has been cut
75. Nurse Joanna did not stop by simply drying the baby thoroughly upon birth , she observed other details as
essential part of the immediate care of a normal newborn which included:
a. Skin to skin contact followed by placing the baby in a warming incubator
b. Removing used wet cloth, and covering the baby with clean, dry cloth
c. Stimulating the baby by slapping the soles of the babys feet
d. Deep suctioning of the airway to remove mucus
76. In further applying essential newborn care (ENC), nurse Joanna keeps in mind that care of the umbilicus
should include:
a. Cleansing with cooled : boiled water
c. Covering with a sterile compress
and leaving umbilicus uncovered
d. Cleansing with alcohol
b. Applying antibiotic cream
77. Administration of Vitamin K to the newborn is necessary since
a. Newborns have no intestinal bacteria
b. Hemolysis of the fetal red blood cells destroys vitamin K
c. The newborns liver is incapable of producing sufficient vitamin K yet
d. Newborns are susceptible to avitaminosis
78. Cord clamping and the traditional milking of the cord immediately post delivery have now proven to be nonbeneficial and may result in causing more harm and complications especially in preterms and fragile blood
vessels in the brain of the newborn. NOW new practices have been introduced as part of essential
newborn care termed as
a. Properly timed Cord Clamping
c. Unang Yakap
b. Routine separation
d. Partographing
e.
f. Situation 8: In the community nurses serves a pivotal role in the process of communication
exchanges between clients and the providers of care. The following events are evidences of
these.
79. In a prenatal visit of Rebecca to your family Care clinic you noted her saying.During the first 3 months of my
pregnancy I have been very nauseated and I do not seem to understand why? Which may be your first
response?
a. The nausea you experience is due to an increase in the basal metabolic rate brought about by your
pregnancy
b. The nausea you experience is caused by the secondary influx of hormones produced by the
endocrine system
c. If you different kinds of food you will not be nauseated
d. Fatigue as a result of your pregnancy brings about your nausea
80. On a Monday morning at your Family Care Nursing Clinic, a mother of a 2 year old postoperative child and
has just been discharge from the hospital came appearing to be overly anxious and tells you .My child
seems so restless. Your FIRST course of action should be
a. Try taking the child for a walk
b. Check the Childs vital signs
c. Check and see when the child had his last pain medication
d. Check if the Childs sutures are still intact
81. You kept that all Fridays are Family Care nursing clinic HOLIDAYS. on Fridays you prefer to do personal
visits You follow-up patients in their homes , do volunteer health work in school or visit industries within the
vicinity. In a student health clinic a client confides to you that her boyfriend informed her that he tested
positive for hepatitis B. Which of the following is your BEST response?
a. You will receive the hepatitis B immune globulin (HBIG)
b. Have you had sex with your boyfriend
c. That must be real shocking to you
d. You should also be tested for hepatitis B
82. An eye surgeon aware of your Friday visit Programcalled on you one day and referred a 4 year old client
whom he scheduled for an eye surgery. When you met her mother she asks how best to prepare her
daughter for eye surgery. Which of the following actions would be best?
a. Draw a picture of the eye and explain what will happen during surgery
b. Help the mother explain to the child how to get ready for surgery using dolls
c. Instruct the mother to tell her daughter that the surgery will only take an hour
d. Guide the mother to read to her daughter an age-appropriate illustrated book about eye surgery
e.
15

f.
g.
h.
83. You are back on your usual Nursing Clinic duties. It is a Tuesday and a woman at 38-weeks gestation
comes to you with problems of vaginal bleeding . which of the following remarks , if made by the client
would suggest PLACENTA PREVIA as potential cause of bleeding?
a. I feel fine, but the bleeding scares me
b. Ive been experiencing severe abdominal cramps
c. I feel nauseated more during the past few weeks
d. The bleeding started after I carried 4 bags of groceries
e.
f. Situation 9: The Philippines witnessed the rise in the incidences of DENGUE in various
parts of the country starting 2009. The following situations affecting children apply
84. In a Barangay in Iloilo where there were high incidences of malaria, a child was brought by her mother
with on and off feeling of dryness and warm skin, temperature reached 37.50C and above, the child has
no general danger signs, with stiff neck, no runny nose, no measles and no other obvious causes of
fever. The child may be classified as having
a. Malaria
c. Fever: malaria unlikely
b. Fever: no malaria
d. Severe febrile disease/malaria
85. It is understood that if a child were living in a no malaria risk areabut who has presenting signs of stiff
neck. The child may be classified as having
a. Severe febrile disease
c. Malaria
b. Fever no malaria
d. Severe malaria
86. You attended to a 3 year old child with measles and with eye complications. this child should be treated
with the following except:
a. Apply gentian violet
c. Give vitamin A
b. Apply tetracycline ointment
d. Follow up in 2 days
87. In classifying dengue cases, which of the following is NOT possible classification of dengue
hemorrhagic fever?
a. None of these
c. Dengue hemorrhagic fever: unlikely
b. Severe dengue hemorrhagic fever
d. Dengue hemorrhagic fever
88. You are attending to 5 cases of dengue in 2 clusters of Barangays assigned to you. Which of the following
treatment modalities SHOULD NOT be considered if the children you are attending to has severe dengue
hemorrhagic fever?
a. Give aspirin
b. Give ORS if there is skin petechiae
c. Apply alternative plan is there is bleeding from the nose or gums
d. Prevent low blood sugar
89.
90. Situation 10: Having a privately owned, community situated nursing care facility makes the nurse
develop rapport with greater number of community residents. She can become a vital community
resource with her nursing skills and knowledge. BUT with the nature of the nurses work, it is
necessary to be conscious about legal considerations.
91. Alvin, a 5 year old boy and his mother are your regular clients in your Nursing Clinic. You accompanied them
to a nearly hospital for referral of what you suspected as an acute case of appendicitis. Alvins parents have
been legally separated for 5 years now but both enjoys joint legal custody. The nurse on duty sought your
assistance in asking the mother for her informed consent for immediate surgery. Together with the nurse on
duty which of the following would be the BEST action?
a. Have mother sign the consent and continue the child preoperative preparation
b. Contact the father to obtain consent
c. Have mother sign the consent and inform surgery
d. Have mother sign the consent and inform the surgeon right away
92. Another of your pediatric patients name Duke got rushed to the hospital with sustained bruises and
lacerations, and a fractured arm. As Duke was being treated in the E.R. his mother requested for you to
come and assist them and the nurse on duty (NOD) was informed of your coming as their family nurse.
Upon arrival you coordinate with the NOD and later you were able to obtain Dukes confession that he got
involved in a frat fight outside school. You and the NOD agreed on which priority actions?
a. Ensure documentation on the Dukes chart
b. Share the information with the hospital social worker
c. Share only this information with fellow health professionals
d. Call for the Police and report the finding
93. You got invited as a speaker in a class nursing students to share your experience and understanding on the
handling of potential legal cases while in practice. Which acts would constitute battery?
a. When you administer an injection to a schizophrenic patient who refuses to take the medication
because he believes it is poison
b. When on doctors order you restrain an agitated patient inside the E.R
c. When you chase a patient who tries to run away taking a walk with you around the hospital
16

d. When you hold the arms of a manic patient who strikes you
94.
95.
96.
97. Nathan, a 10 year old boy and his family are your clients. He was admitted to the hospital for a skin
graft surgery. You went to visit and when you came he is being rolled back from surgery. He is on D5W
infusing into his left arm and you introduced yourself then checked with the nurse on duty (NOD) and
got the following information, he weights 50lbs (23.6 kg) and the physicians order was D5W
2,000cc/24 hours.at this instance you opted to engage the NOD to
a. Set the I. V infusion pump controller to run at 84 gtts/min
b. Monitor the patient for fluid and electrolyte balance
c. Call the physician to clarify the I.V fluid order
d. Ensure accurate records of the patients intake and output
98. You visited one of your family client, a 26 year old mother whose son died of Sudden infant Death
syndrome (IDS) . as you were conversing it is sound for you to
a. Ask how son was positioned in bed while in the hospital at the time of his death
b. Allow the mother to cry and talk about her son and related concerns
c. Ask about her other children at home
d. Explain the cause of SIDS
99.
100.
Situation 11: Mary, a 12 month old infant, was brought to the health center for her regular
well baby check up. Her mother is concerned with her childs growth and development. She
expressed her desire to learn more about this concern
101.
The childs birth weight was 8lbs. Upon assessment the child now weighs 18lbs. In documenting
this result, you know that this weight is:
a. Appropriate for the Childs age
c. Above the expected weight
b. Below the expected weight
d. Individualized and thus unpredictable
102.
In formulating the nursing diagnosis regarding the mothers concern, which of the following
should you consider?
a. Health seeking behavior
c. Knowledge deficit
b. Anxiety
d. Altered health maintenance
103.
In planning care for the infant, you should advise the mother that the best way to help for child
complete the development task for the first year is to
a. Respond to her consistently
b. Expose her to many caregivers to help her learn variability
c. Keep her stimulated with many toys
d. Talk her at a special time each day
104.
To relieve teething discomfort which measure would you suggest an infants mother to use?
a. Provide her with a fluid diet for 2 days
b. Offer her Aspergum to chew
c. Ask her paediatrician for a sedative for her
d. Give her cold teething ring to chew
105.
In evaluating the health teaching on breastfeeding, which of the following observations made by
the mother would reveal correct understanding of breastfeed infants?
a. Breastfed infants usually have fewer stools than bottle fed infants
b. Stools of breastfeed infants tend to have a strong odor
c. Breastfed infants are usually harder than those of bottle fed infants
106.
107.
Situation 12: A nurse even as generalist needs the necessary competencies to attend to
birthing mothers, normal at risk and most especially the high risk in varied settings. The
following conditions apply
108.
Nurse Lucy is a Community Health nurse conducting prenatal care at the Rural Health Unit in
one of the communities in Cotabato. Nurse Lucy is aware that recent developments in Maternal Care
strongly places emphasis on quality antenatal care. The strength of this advocacy is on:
a. Consistent vaginal examination on every prenatal visit
b. Proper care to every pregnant woman providing support to normal pregnancy AND early
detection and management of complications
c. Doing monthly antenatal care during the first 7 months, every two weeks on the 8th month and
weekly on the 9th month
d. Spending more time with every pregnant women especially the high risk ones
109.
On May 12, 2012 a primigravida mother names Mrs. Jenna dela Cruz came to Nurse Lucy for
check up her EDC happens to be May 21. What can you expect to find during assessment?
a. Cervix open, FH-2 fingers below xyphoid process, floating
b. FH at least at the level of the xyphoid process enagaged
c. FH-2 fingers below xyphoid process engaged

17

d. Cervix closed , uneffaced, fundic height (FH) midway between the umbilicus and symphysis
pubis
110.
111.
112.
One Monday morning from a community nursing assignment, Nurse Lucy was transferred to their
RHU attached district hospital. She attends Mrs. Evelyn Datunggalen who is on her seconds vaginal
examination and in labor. Nurse Lucy finds the fetus in the right occiput anterior position and at (-) 1
station. Nurse Lucy based her findings from
a. 1 cm below the ischial spines presentation
b. 1 cm above the ischial spines presentation
c. Presentation in no relationship with the ischial spine condition
d. Presentation directly in line with that of the ischial spines
113.
By 12:00 same Monday another patient attended to by Nurse Lucy in the PHU named Mrs.
Almira del area got admitted in active labor. Nurse Lucy locates the fetal heart rate sounds in the upper
right quadrant of the mothers abdomen. Nurse Lucy would recognize which of the following?
a. This position is referred to as left antero-pelvic
b. Almira will probably deliver very quickly and without problems
c. The fetus is in the most common anterior fetal position
d. Almira will probably have a breech delivery
114.
By 3pm, Nurse Lucy was done with her endorsement when another mother, Mrs. Mercy Shalimar
came in active labor. She trembles, was very tense with contractions and quite irritable, she frequently
says, hindi ko na kaya ito!I can no longer bear this! This behavior may be indicative of the client.
a. Developing some abnormality in terms of uterine contractions
b. Already entering the transition phase of labor
c. As having been poorly prepared for labor in parents classes
d. Needs immediate administration of an analgesic or anethetic
115.
116.
Situation 13: after 10 meaningful years professional practice and using her sayings,
nurse Ruthcelle decides to set up her propriety home healthcare agency. This may be regarded
as another journey towards personal and professional development.
117.
As owner of the agency: nurse Ruthchelle must be concerned about
a. Making her agency yield reasonable return of investments (ROI) for viability and sustained
business growth
b. Finding out if her agency can be exempt from paying taxes
c. Looking for funding support from other business organizations
d. Knowing which government regulatory agencies she should report to
118.
Nurse Rutchelle hires registered nurses to work her as home healthcare nurse whom she
assigns to individuals and families requiring nursing care in their homes. One such care is a 60 year old
client who needs irrigation of his colostomy following surgery for cancer. This case may not qualify for
PhilHealth reimbursement because the client
a. Qualifies as low income client
c. Is under the age of 65
b. Is under home-based care
d. Is not considered handicapped
119.
Nurse Marissa has a caseload of clients whom she needs to see on a regular basis. Which of the
following clients may immediately be admitted as a matter of PRIORITY to a private home healthcare
program?
a. A woman who has delivered a normal healthy infant in a government managed birthing center
b. A client in the terminal stages of cancer at the hospital and will be discharge for home care
c. A family with three(3) family members who tested positive for tuberculosis and just moves into a
barangay
d. A teenage drug abuser who has refused medical treatment but is being seen by a counsellor at
school
120.
A client who had open heart surgery 2 weeks ago and about to be discharge was refrred to
nurse Ruthchelle Home Health Care Agency for daily dressing changes, cardiac rehabilitation and
physical therapy. The assigned home healthcare nurse should
a. Complete the necessary assessment on a weekly basis
b. Develop an individualized plan of care
c. Directly supervise the physical therapy the client receives
d. Provide daily physical care for the client
121.
Nurse Rutchelle plans to present to the local city health board a Stop Smokingcampaign to the
community local public as her social responsibility program. Ruthchelle understands that if the proposal
is accepted, the funding would come from
a. Philanthropic donations
b. Philhealth contribution
c. Local community funds generated from taxes
d. The home healthcare agency owned by nurse Ruthchelle
122.
18

123.
124.
125.
126.
Situation 14: Mrs Neomi Reyes was admitted due to lumbo-sacral pain. She is a G5p3 with
9 weeks AOG. On admission the following findigs were revealed: BOW intact cervical dilatation
4cm, Descent -5 FHT-150/minute. BP-120/80. Pulse rate-60 beats per minute and temperature is
36.8 C. Using this PARTHOGRAPH.
127.
128.
129.
130.
131.
132.
133.
134.
135.
136.
137.
138.
139.
140.
141.
142.
143.
144.
145.
146.
147.
148.
149.
150.
151.
152.
153.
154.
155.
156.
157.
158.
159.
160.
161.
162.
163.
164.
165.
166.
167.
168.
169.
170.
171.
172.
173.
174.
175.
176.
177.
178.
179.
180.
181.
182.
183.
184.
185.
19

186.
187.
188.
189.
190.
a.
b.
c.
d.
191.
a.
b.
c.
d.
192.
a.
b.
c.
d.
193.
a.
b.
c.
d.
194.
a.
b.
c.
d.

You immediately can interpret from the parthograph that Mrs. Neomi Reyes condition as
There is normal progress of labor
Unsatisfactory progress due to cephalo-pelvic disproportion
There is fetal distress
Unsatisfactory progress due to inadequate uterine activity
You recorded a 3+ in the moulding, this means that
The bones are touching is each other
The bones are severely overlapping each other
The bones are overlapping each other
The bones are separated and sutures can be felt easily
Which of the following nursing action should you not include in the care of Mrs. Reyes?
Labor Augmentation/doctors order
Amniotomy
Observe for 30 minutes more for possible caesarean section if no improvement
Continue monitoring fetal heart beat and cervical dilatation
At 1 A.M. what is the frequently and duration of Mrs. Reyes uterine contraction?
Two contraction in 15 minutes lasting more than 40 seconds
Two contraction in 10 minutes lasting more than 40 seconds
Two contraction in 15 minutes lasting more than 40 seconds
Two contraction in 10 minutes lasting more than 40 seconds
The descent of the head at 1 pm is at
Two fingers above the symphysis pubis
Five fingers above the symphysis pubis
Three fingers above the symphysis pubis
The fetal head is engaged

195.
196.
Situation 15: Good and comprehensive nursing assessment among infants and children is an
important aspect of determining appropriate, safe and quality nursing care interventions. The
following apply
197.
A 4-week old infant with symptoms of pyloric stenosis was brought by her mother to your clinic. Which
of the following statements would you expect the mother to make about her sons symptoms?
a. My sons bowel movements have turned black and sticky
b. My son spits green liquid after feeding
c. My son seems hungry all the time
d. I really have to encourage my son to suck the bottle
198.
A teenager comes to your clinic with problems of fatigue, sore throat, and flu-like symptoms in the last
2 weeks. Physical examination levels enlarged lymph nodes and temperature of 37.9 0C. which these
statements do you BEST make?
a. Stay in your room until all of your symptoms are gone
b. Do not share drinking glass or silverware with anybody
c. Eat in a separate room away from your family
d. Cover your mouth and nose when you sneeze or cough
199.
You are caring for a 2 month old infant to which a pH probe test indicated reflux. Which nursing
action is MOST appropriate?
a. Raise the head of infants bed
c. Instruct properly the mother how to do
b. Do not give the next feeding
CPR
d. Keep a normal feeding schedule
200.
You are visiting 3 month old child whom you previously saw in your clinic . he is now on Bryants
Traction for developmental dysplasia of the hips. Which of the following toys would be appropriate for you to
offer the infant to keep him occupied while hospitalized?
a. Colourful plastic : non toxic blocks
c. A stuffed toy animal
b. A toy rattle
d. Nursery rhymes played on tape
201.
One early incoming as you were opening your nursing clinic, a 5 year old boy was rushed to you in an
emergency after ingesting a bottle of baby aspirin. You are to observe the boy for which signs and
symptoms?
a. Tinnitus and gastric distress
c. Nausea and vetigo
b. Dysrrhythmia and hypoventilation
d. Epislaxis and paralysis
e.
f. Situation 16: in some hospitals nurses assigned to care for mothers with reproductive health
concerns are given a TRIAGE assignment. They rotate in the obstetrics unit/ward, the labor
room and delivery room. Your nursing to clients in these units are vital in maintaining
wellness of these women. The following questions apply.
202.
You are taking care of a woman in labor room whose abdomen remains rigid and hard between
contractions. Upon further assessment of fetal heart rate registered at 100/minute. The PRIORITY
PROBLEMS in the care of this client would be
20

203.
be

a. Risk for fetal death


c. Ineffective breathing
b. Alteration in comfort
d. Fluid electrolyte imbalance
e.
f.
g.
During vaginal examination , a prolapsed cord is assessed, your PRIORITY intervention would

a. Instruct the client no to push during contractions


b. Ask relatives to leave the room for privacy
c. Prepare client in Trendelenberg position
204.
Of the following endorsed to you by the outgoing nurse on duty, who would you assess
immediately? The client who
a. Exhibits early deceleration on the fetal monitor
b. Is upset because her obstetrician in on vacation
c. Is 10 cm dilated and 100% effaced
d. Cannot decide if she wants anesthesia during delivery
205.
You are assigned to the postpartum unit/ward and you are to take care of several clients when
prioritizing care for these client you will first assess the client who
a. Is upset because the baby does not nurse when breast feed
b. Has saturated several sanitary napkins during the night
c. Refuses to have the newborn room in with her
d. Complains of pain and burning sensation upon urination
206.
Your client in labor is diagnosed with pre-clampsia. Which interventions would be most
appropriate for this client?
a. 1. Check current intravenous administration of magnesium Sulphate
b. 2. Frequently check clients telemetry monitor
c. 3. Assess the clients deep tendon reflexes
d. 4. Inform the nursery for incoming infant
e. 5. Administer Furosemide (lasix) intravenously as ordered
f. 2,3,5
h. 2,4,5
g. 1,2,5
i. 1,3,4
j.
k. Situation 17: Care of children, well at risk, or even at high risk conditions places
overwhelming responsibilities to nurses. The following conditions apply
207.
A couple brought with them their 7 year old girl diagnosed as having pituitary dwarfism. The
couple seen anxious and was looking for some kind of emotion and psychological support. In your
assessment , you expected which clinical manifestations to be MOST likely to be evident.
a. Abnormal body proportions
c. Delicate features
b. Course, dry skin
d. Early sexual maturation
208.
A 14 year old girl whose parents have been consulting with you in your Family care Nursing
Clinic (FCNC) was admitted to the hospital for treatment of 2nd and 3rd degree burns sustained from a
house fire. You visited them in the hospital and noted an IV infusion started over the girls left forearm.
Instantly what comes to mind is that the primary purpose of this IV is to
a. Maintain fluid balance
c. Provide a route for pain medications
b. Prevent G.I upset
d. Obtain blood specimens for analysis
209.
A toddler name Peter, whose parent were also consulting in your FCNC had lead poisoning and
was rushed to the hospital. There was an order to encourage fluids and the same were relayed to the
parents. When you visited the parents asked what kind of fluids are best given, and as a family nurse
coordinating closely with the nurse on duty (NOD) you reiterated that it is best to give
a. Fruit juices
b. Orange juice
c. Water
d. Milk
210.
A 34 year old single mother dropped by your FCNC and tells you that she always have difficulty
forming relationships. The mother conveyed to you the message that she is worried that her 7 year old
daughter might have the same problem later. Of the following statements which do you think is BEST
to make?
a. Children develop trust from 6-12 of age
b. Children develop trust from birth to 18 months of age
c. Children develop trust from 18 months to 3 months of age
d. Children develop trust from 3-6 years of age
211.
Again at your FCNC a couple came to you with relating problems relative to the care of the
newborn with fetal alcohol syndrome. Which of the following should be reiterated as important
considerations by the parents
a. Provide feedings via gavage to decrease energy expenditure
b. Decrease touch to prevent overstimulation
21

c. Replace vitamins depleted as a result of poor maternal diet


d. Prevent iron deficiency anemia
212.
213.
214.
215.
216.
217.
Situation 18: You are the nurse assigned in the OB-GYNE Unit of the hospital in your
Provincial Hospital. The following conditions and situations apply
218.
A client named Rebecca; gravid 2 para 1 was admitted with complaints of hypertension. She
even remarked that her wedding band was tight. In doing assessment which of the following direct to
the entertain early pre-eclampsia./
a. Pain in her epigastrium together with headache
b. Swelling of her face with proteinuria
c. Blurring of vision and proteinuria
d. Hypertonic reflexes together with polyuria
219.
You also have Jinky who is on her 4th Stage of Labor. Where do you palpate the fundus?
a. 2 cm above umbilicus
c. To the right of the umbilicus
b. 3 cm below umbilicus
d. At the umbilicus
220.
Another patient a 21 year old woman who is at her 16 weeks gestation and underwent
amniocentesis. Her name is Eva. She asked you what her OB doctor intends to find out from the
procedure. You appropriate response is based on the understanding that which of the following
conditions may be detected through this test?
a. Cleft lip palate
b. Talipes equinovarus
c. Tetralogy of Fallot
d. Hemolytic disease of the newborn
221.
Doctor Rodolfo Rodriquez is attending to a 43 year old woman whom he had confined for
observation and whom he just placed on estrogens (premarin) 0.625mg OD. Learning about this
treatment regimen you are aware of the initial side effects of this medication which is?
a. Nausea
c. Visual disturbances
b. Tinnitus
d. Ataxia
222.
You are caring for Nympha, a multipara client who just delivered a female infant one hour ago,
you observed that Nymphas breasts are soft, the uterus boggy, to the right of the midline, and 2 cm
below the umbilicus; moderate lochia rubra. What action is called for you to undertake with these
findings?
a. Perform a straight catheterization
c. Put her baby to her breast
b. Massage her uterine fundus
d. Offer bedpan immediately
e.
f. Situation 19: The nurse is involves in community organizing activities of Barangay
Malusog. The following questions pertain to this situation.
g.
223.
The nurse makes an ocular survey of Barangay Malusog. Upon reaching the barangay it is
appropriate for her to
a. Pay a visit to the barangay captain
d. Assemble community members for a
b. Encourage community involvement
meeting
c. Introduce self to the community
224.
Carrying out the activities involving maximum community participation is referred to as
a. Mobilization
c. Community organization
b. Integration
d. Community assembly
225.
To ensure collective participation of the people , community organization should be formed
through a community assembly. The purpose of having community assemblies are the following ,
EXCEPT
a. Gain cooperation , pledge support and work together on their own
b. Help people identify their own problem
c. Plan for the solution of identified problems
d. Undertake measures to solve existing problems of the community
226.
Which of the following approaches is the BEST in applying community organization in
Community Health
a. Primary Health Care
c. Community Oriented Organization
b. Universal Health Care
d. Community Development
227.
Community organizing ends when the community is already self reliant. This signals that the
community organizers are now ready to pull out of the community because
a. Organisers can expand to other poor communities
b. It can evaluate the outcomes of the programs
c. It indicates the community organization is finished
22

d. It will prevent dependency of the community


228.
229.
230.
231.
232.
233.
234.
235.
236.
Situation 20: It is true that health education is very much great part of our nursing
responsibilities; the same should be evidently practice with birthing mothers. The following
conditions speak of these responsibilities.
237.
238.
A diabetic mother named Nemia plans to breastfeed her baby. You explained that, if maffle is
hyperglycaemic.
a. The production of milk may be impaired
b. Her baby will receive insulin in the milk
c. Her baby will not grow well
d. The glucose content of her breast milk may be high
239.
Jennifer has successfully adapted in her let-down reflex and ably breastfeeding, but preventing
the occurrence of cracked nipples now becomes one of your nursing concerns Shine should therefore
be taught to
a. Wash her nipples with water only
b. Nurse at least 20 minutes on each breast the first day
c. Use plastic bra liners
d. Apply lanolin prior to feedings
240.
Mrs. Jie Agom just delivered her first baby and whom she is expected to breastfeed , in the
development of your teaching plan which of the following instruction must you include?
a. Try to schedule feedings at least every 3 to 4 hours
b. Wash nipples with soap and water before each feeding
c. Avoid nursing bras with plastic lining
d. Supplement with water feedings when necessary
241.
Lilias prenatal antibody titer shows that she is not immune to rubella and will receive the
immunization after delivery. You would include which of the following instructions in your teaching plan?
a. Breastfeeding should be postponed for 5 days after the injection
b. Another immunization should be administered in the next pregnancy
c. Pregnancy must be avoided for next 3 month
d. An injection will be needed after each succeeding pregnancy
242. Jasmine had a normal vaginal delivery 12 hours ago and is to be discharge from the birthing center.
You evaluated that Jasmin understands the teaching related to the episiotomy and perineal area when
she states
a. The ice pack should be removed for 10 minutes before replacing
b. The anesthetic spray, then the heat lamp, will help a lot
c. The water for the Sitz bath should be warm, about 102-1050F
d. I know the stitches will be removed at my postpartum clinic visit
243.
244.
245.
246.
247.
248.
249.
250.
251.
252.
253.
254.
255.
256.
257.
258.
259.
260.
261.
262.
263.
23

264.
265.
266.
267.
268.
269.
270.
271.

272. A1 PASSERS TRAINING, RESEARCH, REVIEW AND DEVELOPMENT COMPANY


273.
PRE-BOARD EXAMINATION
274.
275.
NURSING PRACTICE III-Care of Clients with Physiologic and Psychosocial Alteration
276.
277.
278.
GENERAL INSTRUCTIONS:
279.
1. This test booklet contains 100 test questions.
280.
2. Read INSTRUCTIONS TO EXAMINEES printed on your answer sheet.
281.
3. Shade only one (1) box for each question on your answer sheets. Two or more boxes shaded will
invalidate your answer.
282.
4. AVOID ERASURES.
283.
284.
INSTRUCTIONS:
285. 1. Detach one (1) answer sheet from the bottom of your Examinee ID/Answer Sheet Set.
286. 2. Write the subject title Nursing Practice I on the box provided.
287. 3. Shade Set Box A on your answer sheet if your test booklet is Set A; Set Box B if your test booklet is
Set B.
288.
_________________________________________________________________________________
_________
289.
290.
Situation 1: Upon discharge, the patient with Chronic Obstructive Pulmonary Disease
(COPD)requires considerable patient and family teaching.
1. A nurse instructs a client diagnosed with COPD to use pursed-lip breathing. The client inquires the
nurse about the advantage of this kind of breathing. The nurse answers , that the main purpose of
pursed lip is to
a. Prevent bronchial collapse
c. Achieve maximum inhalation
b. Strengthen the intercostals muscle
d. Allows air trapping
2. The nurse teaches a patient about the use of respiratory inhaler. Arrange the steps in using an inhaler
chronologically
3. 1. Press the canister down with your fingers as you breathe in
4. 2. Wait one minute between puffs if more than one puff is prescribed
5. 3. Inhale the mist hold your breath at least 5 to 10 seconds before exhaling
6. 4. Remove the cap and shake the inhaler
a. 4,1,2,3
c. 4,1,3,2
b. 3,4,2,1
d. 1,2,3,4
7. The physician prescribed monitoring closely of clients oxygen saturation of the blood which of the
following will you prepares?
a. Electrocardiogram machine
c. Spirometer
b. Pulse oximeter
d. Blood pressure apparatus
8. Patients suffering from COPD are taught to avoid shifts to temperature and humidity. It should be
emphasized that heat increases boy temperature and thereby raising the
a. Risk for infection
c. The oxygen requirements
b. Fluid intake
d. Anxiety level
9. COPD patients may be taught the following pulmonary hygiene measures to improve clearance of
airway secretion, EXCEPT:
a. Measure fluid intake
c. Postural drainage
b. Effective coughing
d. Complete bed rest
e.
f. Situation 2- Potential environment hazards exist in various modalities in the Operating
Room that may affect the well being of the client and health care workers. it is the
responsibility of the perioperative nurse to maintain a positive environment for all
concerned.
10. The circulating nurses are aware that many factors combine to contribute hypothermia or hyperthermia
in the surgical patients. Below are options that the perioperative nurse can adopt to maintain desired
temperature. Select all that apply:
a.
1. Adjusting the OR suite temperature
b.
2. Limiting area of skin preparation and surface surgical draping

24

c.
3. Applying warm blankets to clients upon arrival in the OR and after sterile drapes have been
removed
d.
4. Keeping the OR door closed throughout the surgical procedure
e.
5. Limiting the exposed skin area during positioning
f.
6. Placing rolled linen on both sides of the client during skin preparation to catch excess water.
g. All except 2 and 5
h. 1,2,4 and 5
i. 1,2,3 and 6
j. All of these
k.
l.
m.
11. During transport of postoperative clients, which of the following would you NOT recommended to be
adopted?
a. Pushing the patients feet first avoiding rapid movement on the hallways and corners
b. Hanging and securing I. V. Containers over the clients head
c. Elevating the side rails and using safety straps
d. Keeping the client warm with blanket
12. The surgeon of a client for Dilation and Curettage (D & C) who is on triple Anti TB drugs complained
why his case is scheduled last for the day. The OR nurse offers which of the following BEST reason?
a. The case is relatively short and easy
b. Your anaesthesiologist preferred this time slot
c. Foremost , we considered the safety of other clients
d. There is no emergent need to do the case ahead of the other schedule
13. You are preparing case assignment for the following day, which of the following assignment is SAFE for
nurse Kamille who is one her 1st trimester of pregnancy?
a. Cast room with x-ray facilities
b. Endoscopy room where clients are given intravenous mild sedation
c. Billroth 11 under general anesthesia
d. Laparoscopic cholecystectomy
14. Before the end of the shift, waste management was discussed. The different kinds of waste and their
proper disposal were presented. The following falls under the pathologic waste category, EXCEPT:
a. Patients personal belongings
c. Blood and body fluids
b. Amputated limbs
d. Specimens
e.
f. Situation 3-Mrs. Magsingal , a 47 year old teacher sought admission to the hospital
hemorrhage due to diverticulitis
g.
15. Nurse Ellen read the order of the attending physician Start Blood transfusion of 2 units compatible
blood to run for four hours each unit. Monitor closely and report untoward reactions. The nurse
prepared for cross matching. Which of the following is an APPROPRIATE action of the nurse?
a. Obtain blood transfusion set from the supply room
b. Have blood sample extracted by the medical technologist start intravenous infusion of normal
saline solution
c. Start intravenous infusion of normal saline solution
d. Call blood bank for the prescribed blood
16. Nurse Ellen is successful in collaborating with the test of the team if she understands that the
diverticulum may bleed due to
a. Erosion of the adjacent blood vessel
c. Prolonged constipation
by a fecalith
d. Severe inflammation of the sigmoid
b. Irritable bowel syndrome
17. Additionally, the physician ordered to asses for evidence of lower intestinal bleeding the nurse does
which of the following?
a. Visual examination of the stool
c. Gualac testing of stool for occult
b. Accurate measuring hourly output
blood
d. Routine stool examination
18. When collaborating with the health team members , which of the following BEST describe the nurse role?
a. Shares and implements doctors order effectively
b. Listens to the different views of the significant other.
c. Helps client understand the treatment plan
d. Encourages participation of the client in the total care
19. Total parenteral nutrition was started. The nurse understands that the caloric nutrient content and
amount of this intervention is determined by the following EXCEPT
a. Physician
c. Nutritionist
b. Pharmacist
d. Nurse
e.
25

f. Situation 4- Mr. Sta Ana, a post acute myocardial infarction (AM) on his 2nd day post attack
is assigned to you. The physician said his recovery is uneventful. The following
questions apply
20. Morphine sulfate intravenous (I. V) was prescribed for pain. The nurse understands that morphine
sulfate I.V was preferred because of two reasons. these are
a. 1. Rapid onset of action
b. 2. Elevates enzyme levels
c. 3. Increases cardiac output
d. 4. Bypass the variable rates of absorption
e. 2 and 3
g. 1 and2
f. 3 and 4
h. 1 and 4
21.
22.
23. Mr. Ana is taking Aspirin, a platelet inhibitor. The client understood the nurses instruction on how to take
the drug if he
a. Swallowed medicine with a small amount of water
b. Took the medicine with meals
c. Chewed and allowed the drug to dissolve with saliva
d. Took the medicine two hours before meals
24. The clients wife observes the facial expression of Mr. Sta. Ana and interprets that her husband is in
pain she ask the nurse . What is causing the pain?The nurse responded that:
a. Release of tissue substance during inflammatory process can stimulate pain receptors
b. Pain is felt when the myocardial muscles contract rapidly
c. Pain is triggered by high blood pressure
d. Chest pain occurs when the oxygen demand of the heart muscles is not met
25. One of the priority nursing diagnoses is Ineffective tissue perfusion. Which of the following would you
watch for as watch for as FIRST indication of altered perfusion?
a. Adventitious lung sounds
c. Change in level of consciousness
b. Abnormal heart sounds
d. Presence of dysrhythmias
26. During episodes of chest pain, which of the following procedures would the nurse expect to be
prescribed to provide assessment for myocardial infarction?
a. Echocardiography
c. Radionuclide imaging
b. Electrocardiography
d. Angiography
e.
f. Situation 5-A group of nursing students decided to conduct a phenomenological research
project on the lived experience of being isolated among patients who have experienced
isolation as a technique in infection prevention.
27. In a phenomenological research, the research team understands that the experience of being isolated
will be:
a. Interpreted by the participant for the researcher
b. Explained by the researcher to the participant
c. Interpreted by the researcher for the participant
d. Interpreted by the researcher with the participant
28. When determining adequate number of participants, the research team will consider which of the following?
a. Sample size of the participant will be determined before the study
b. Number of participants will be adequate when data obtained are saturated
c. Participants should be representative of the target population
d. The convenience type of sampling will be the most appropriate sampling method
29. The research team planned to utilize audio-recorded interviews as their method of collecting data. Which of
the following techniques would ensure that data obtained will be adequately analyzed?
a. 1. Simultaneously listen to the recording and read the written transcript , then note observations
b. 2. Repeatedly listen to the audio tape recordings
c. 3. Make notations while listening to the audio-tape recordings
d. 4. Transcribe audio-recorded interview word to word
e. 1 only
g. 1 and 4
f. All of these
h. 1,3 and 4
30. After the research team has dwelled with the data the team decided to conduct data reduction. Which of
the following is the team expected to do?
a. Classify data based on a theoretical perspective
b. Eliminate data which are not related
c. Break text down into subparts and label accordingly
d. Identify patterns in the obtained data
31. During the interpretation phase, the research team is expected to answer which of the following questions?
a. What do you findings show?
b. What is going on?
c. What is the meaning of the experience of isolation?
d. Are the themes justifiable and grounded from the data?
26

32.
33.
34.
35.
36.
37.
38.

Situation 6- You are caring for a client who is with nasogastric tube (NGT) for feeding.
When assessing for NGT placement, which three methods are often recommended?
1. Measurement of Exposed tube length
2. Visual assessment of aspirate
3. Auscultation method after air injection
4. pH measurement of aspirate
a. 2,3 and 4
c. 1,2 and 3
b. All except 2
d. 1,2 and 4

39.
40.
41.
42.
43. It is important to maintain patency of the nasogastric tube. The tube is irrigated every 4 to 6 hours.
Which solution would you use?
a. Tap water
c. Normal saline
b. Bottled water
d. Lactated Ringers
44. When giving tube feedings and medications, which position of the client will reduce risk of reflux and
pulmonary aspiration?
a. Supine position with one pillow supporting the head
b. Semi- Fowlers position with the head elevated from 30-45
c. Supine position with the head turned to one side
d. Dorsal recumbent
45. When giving single compressed tablet medication by NGT, it should be crushed and dissolved in water.
How would the nurse APPROPRIATELY administer enteric coated tablet?
a. Let the client swallow the tablet as is
b. Pulyurize the tablet finely to change the tablet form
c. Request the pharmacist to change the tablet form
d. Crush and dissolve in distilled water
46. Diarrhea is a one of the most common complications of tube feeding. Which of the following nursing
actions will prevent this complication?
a. Administer feeding by continuous drip rather than bolus
b. Give high fiber formula
c. Dilute formula to half the concentration strength
d. Instil liberal amounts of water to flush the tubing before and after feeding
47.
48. Situation 7- Appropriate and ethical nursing practice should always respect the patient right in
any health care setting. The following questions apply:
49. With the advancement of information technology, the nurse understands that breach of confidentiality
can happen LEAST in which of the following scenario?
a. Keeping the X-ray plate hanging in the negatoscope
b. Clients laboratory results are transmitted to the patient care unit through a Hospital
Computerized System
c. Allowing telephone orders as means to transmit doctors order
d. Patients hospital account viewed is computers placed in the hospital corridors
50. When restoration of health is no longer the goal of care and end-of-life care is the goal. Artificial
nutrition and hydration can be prepared and continued to be administered. The nurse can administer
artificial nutrition through the following avenues EXCEPT:
a. Intravenous (I.V.) infusion
c. Ileostomy tube
b. Nasogastric tube
d. Gastrostomy tube
51. Nurse Sofia is in charge of an elderly client with chronic severe COPD with complications . she recalls
that hospice care might be of benefit to the client . Which of the following statements is TRUE about
hospice care?
a. In hospice care, practical support is provided based on the wishes of a client and needs of the
family
b. Health care workers are not offering hospice care because they dont like clients to think that
they are giving up on them
c. Hospice care is a part of normal life and provides support for dignified individuals
d. Hospice care concept leads to think that they are hopeless cases
52. Nurse Sofia is about to request an elderly client with emphysema to sign the consent for thoracostomy
but assesses the client as incompetent . with the client in the hospital , is a 15 year old boy watcher.
Which of the following options would be MOST ethical for the nurse to follow?
a. Send the Boy watcher to fetch the clients next of kin immediately
b. Call the client next of kin right away
c. Refer to the attending physician
d. Inform the head nurse
27

53. Health care providers need always to point out to clients and significant others that the order of Do
not resuscitate (DNR) means the following EXCEPT:
a. Allow natural death to happen (AND)
c. Food and food supplements are
b. Comfort measures are withheld
sustained
d. No heroic measures done
e.
f. Situation 8- Editha , 22 years old, was bought to the hospital by her mother for chief
complaints of pallor, shortness of breath and weakness. The doctors impression was
anemia.
54. The nurse knows that the BEST areas to assess pallor that are characteristics of anemia are the:
a. Conjunctivae and lips
c. Lips and fingernails
b. Palms and fingernails
d. Tongue and fingers
55.
56.
57. To establish a diagnosis the nurse would expect the following laboratory tests to be ordered by the
physician EXCEPT:
a. Iron studies
c. Bone marrow aspiration
b. Complete blood count
d. Erythrocyte sedimentation rate
58. Based on the initial assessment, the nursing diagnosis identified Activity intolerance related to
weakness and shortness of breath. Which of the following is the MOST relevant nursing intervention?
a. Passive regular exercise to lower
c. Auscultate lungs for abnormal breath
extremities
sounds
b. Change position every two hours
d. Maintain on high Fowlers position
59. The nurse was instructed by her senior to stay with the patient for at least 15 minutes after initiating
blood transfusion PRIMARILY because of which of the following reasons?
a. Vital signs must be monitored every
c. Transfusion reactions can occur
15 minutes
d. It is a nursing order
b. Patient needs assistance
60. When the nurse checked the patients vital signs 15 minutes after blood transfusion was initiated, the
nurse observed the blood pressure to have increased. The nurse suspects which of the following to
have occurred?
a. Blood transfused is contaminated
b. Circulatory system could not accommodate blood volume transfused
c. Infusion of incompatible blood products
d. Hypersensitivity to the donors plasma proteins
61.
62. Situation 9- indicators are essential in the measurement and monitoring of quality health care.
The nurse plays a key role in the consistent implementation of standards of care in any unit in
the hospital. The following questions apply
63. You admitted Laura, a 26 year old mother, from the Emergency Room for emergency appendectomy.
Which of the following will you use to PROPERLY identify the client?
a. Request the client to state her name and the complete name of her physician
b. Ask the companion to state the clients name and address
c. Ask the client to state her name and birthday
d. Read the name of the client from the chart and name tag
64. You are do initial assessment. Which STANDARD PRECAUTION guideline should be observed?
a. Greet the client by her first name
c. Observe hand hygiene
b. Introduce yourself
d. Drape the client appropriately
65. It is medication time. The nurse is aware that distraction any phase of drug administration can cause
errors. Select which strategy can give MOST protection while preparing the medication?
a. Don a medication vest with visible warning Dont Disturb.
b. Put your cell phone on silent mode
c. Inform your co-staff that you are going to prepare medications for your clients
d. Prepare medication at the bedside
66. Upon entry to the Operating Room, the Client was properly identified. Correct site and procedure shall
likewise be identified by which of the following?
a. Nothing proper markings and endorsement
b. Allowing the client and companion to identify the site and procedure
c. Asking the client and reading the chart
d. Verifying from the OR schedule and patients chart
67. Personal Protective Equipment (PPE) like the face mask when worn correctly offers maximum
protection against transmission of droplets. Below are descriptions of how a mask is worn. Which
statement describes a CORRECTLY WORN MASK?
a. With the two loops on either side of the mask, anchor it to both ears and adjust to cover the nose
and the mouth
b. Place the mask to cover the nose and mouth with two ties tied at the back of the head
28

c. The mask is placed to cover the nose and mouth with the four sides snugly fitting against the
skin and with two pairs of ties tied separately at the back of the head
d. The mask is placed to cover the nose, ear and mouth tied at the back of the head
68.
69. Situation 10-Ginger, 45 years old male, diagnosed with gastric cancer was admitted to the Post
Anesthesia Care Unit (PACU) post partial gastrectomy and gastrojejunostomy. Though still
sedated patient responds to commands.
70. The nurse who admitted the patient performed an initial head to toe assessment. Which of the
following assessment is done FIRST by the nurse?
a. Assess level of consciousness
c. Observe general appearance
b. Determine level of discomfort and
d. Take vital signs
pain
71. When the patient has been stabilized , the PACU nurse transferred the patient to a private room, while
endorsing, the receiving nurse in the private room performed her initial assessment and noticed that the
nasogastric tube of the patient tube of the patient was out of place. Which of the following will the
receiving nurse do FIRST? She will:
a. Remove the NGT and report to the surgeon
b. Secure the NGT with tape and refer to the surgeon
c. Ask the PACU nurse to validate her observation
d. Document observation and report to the surgeon
72. The nurse understands that the immediate postoperative intervention that PREVENTS disruption of the
gastric suture lines post partial gastrectomy and gastrojejunostomy is maintaining:
a. Pressure dressing
c. Complete bed rest
b. Fluid and electrolyte balance
d. Nasogastric tube to drainage system
73. In the nursing care plan, the nurse identified dumping syndrome as a potential problem when patient
starts to take clear liquids. Which of the following symptoms should the nurse watch as a result of
peristaltic stimulation?
a. Nausea and vomiting epigastric pain, borborygmus
b. Tachycardia, diaphoresis, hypoglycaemia
c. Abdominal cramping, light headedness , confusion
d. Orthostatic hypotension, dizziness, palpitations
74. To promote adequate nutrition, which of the following interventions would be LEAST helpful for the
client?
a. Liquids and solids are taken at separate time
b. Meals should be small and more frequent
c. Assume recumbent position for 30 minutes post meal
d. Increase intake of carbohydrates
e.
f. Situation 11- Marissa, a 21 year old college student was admitted at 12 noon because of a
generalized abdominal pain which became localized after midnight on the right lower
quadrant accompanied by nausea and vomiting. In the Emergency Department the
diagnosis of Acute appendicitis was confirmed. Marissa was scheduled for
Appendectomy.
75. The developing of appendicitis usually follows a pattern that correlates with the clinical signs. The admitting
nurse understands that the appendix initially becomes distended with fluid secreted by its mucosa following:
a. Fibrotic changes in the inner walls of the appendix
b. Obstruction of the appendiceal lumen
c. Impairment of blood supply to the appendix
d. Proliferation of microorganisms inside the appendix
76. The physician noted upon palpation of the McBurneys point localized and rebound tenderness. Which of the
following demonstrates this observation?
a. Pain aggravated by coughing
b. Pain increased with internal rotation of the right tip
c. Rigid boardlike abdomen
d. Relief of pain with direct palpation and pain on release of pressure
77. Preoperative nursing care plan includes Potential complications related to ruptured appendixas one of the
nursing diagnosis. Which of the following is the nurse expected to report immediately as a possible sign of a
ruptured appendix?
a. Severe nausea and vomiting
c. Sudden increase in body temperature
b. Unbearable excruciating localized pain
d. Pain subsides
78. To prevent perforation of the inflamed appendix, which of the following will the nurse consider as an effective
intervention?
a. Keep on NPO
c. Maintain on complete bed rest
b. monitor progress of pain
d. Apply hot compress to abdomen
79. Postoperative medical diagnosis of the client is Perforated appendix. Client has a nasogastric tube
connected to continuous drainage. Which of the following is the purpose of this intervention?
a. Medium to cleanse the upper GI tract
29

b. Relieve pain due to abdominal


c. Drain out blood
distention
d. Intestinal decompression
e.
f. Situation 12- You are assigned in the medical unit and assigned to take care of 5 patients with
various cardiovascular conditions. One of your initial activities is to gather date about your
patients.
80. During a physical examination of Anna, you palpated the PMI (point of maximum impulse) in the 5 th
intarcostal space lateral to the midclavicular line. Which of the following is the MOST appropriate action for
you to do?
a. Assess Anna for symptoms of the left ventricular hypertrophy
b. Ask Anna about risk factors for coronary artery disease
c. Auscultate both the carotis arteries for a bruit
d. Document that the RMI in the normal location
e.
f.
g.
h.
81. You are admitting a new patient, Bernie, for coronary artery disease and started to obtain his health
history. Which of the following questions would you use when obtaining subjective data related to
Bernies health perception health management functional health pattern?
a. How often do you have your cholesterol level and blood pressure checked?
b. Have you had any episodes of fever, sore throat, or streptococcal infections?
c. Are there any symptoms that seem to occur when you are feeling very low?
d. Do you ever experience any discomfort or indigestion resulting from exercise or activity?
82. While doing a physical examination on Maria who is in thin 72 year old patient, you observe pulsation of
the abdominal aorta in the epigastric area just below the xiphoid process. You inform Maria that this is:
a. An indication that an abdominal aortic aneurysm has probably developed
b. Related to normal elevated systemic arterial pressure
c. Most likely due to age related sclerosis and enelasticirt of the aorta
d. A normal assessment finding for a thin individual
83. While doing assessment on Grace, who has heart failure , you note that she has jugular venous
distension (JVD) when lying flat in bed, your next action will be to:
a. Palpate the jugular veins and compare the volume and pressure on both sides
b. Use a centimetre ruler to measure and document accurately the level of the JVD
c. Elevate Grace gradually to an upright position and continue to examine the JVD
d. Ask her perform the valsalva maneuver and observe the jugular veins
84. A nursing student is assigned to Lucia, who is one of your patients, and she is doing a physical
assessment. You will need to intervene IMMEDIATELY if the nursing student:
a. Palpates both carotid arteries simultaneously to compare pulse quality
b. Uses the palm of the hand to assess extremity skin temperature
c. Places Lucia in the left lateral position to check for the PMI
d. Presses on the skin over the tibla for 10 seconds to check for edema
e.
f. Situation 13-Nurse Daisy has three discharged client during the early part of the P. M.
shift. The senior nurse assigned her to take charge of any admission during the shift. A
lethargic female client came in for thyroid work-up
85. As prescribed by the attending physician, the nurse instructed the client to undergo radioactive iodine
uptake test the following morning. The client asks to be educated on the test. Nurse Daisy would
explain that the purpose of the test is to:
a. Demonstrate the extent of damage /compresson rendered by the nodule to the trachea
b. Detect if the thyroid nodule is malignant or benign
c. Determine the functional activity of the thyroid gland and differentiate pituitary from thyroid
function
d. Measure the ability of the thyroid gland to remove and concentrate iodine from the blood
86. Before the radioactive iodine uptake test, the nurse should verify which of the following would affect the
result of the test?
a. Over the counter drug intake
c. Height and weight
b. Sleeping habits
d. Food preference
87. The doctor prescribed levothyroxine 0.15 mg. per orem daily after the diagnosis of hypothyroidism was
confirmed. Nurse Daisy administers the medication at which time to obtain the drug optimum the
therapeutic Level?
a. In the morning before breakfast
c. At various times of the day
b. At the patients most convenient time
d. Before bedtime
88. Nurse Daisy included in her health instruction about foods that inhibit thyroid secretions identify these
fruits/vegetables.
89. 1. Spinach
90. 2. Cauliflower
30

91. 3. Squash
92. 4. Raddish
93. 5. Strawberries
94. 6. Guavas
a. All except 3 and 6
c. All except 2 and 4
b. All of these
d. All except 1 and 5
95. Nurse Daisy would include in her discharge plan for the client and significant others the regular intake
of which product that would insure iodine intake?
a. Lugols solution
b. Seafood
c. Warm salt solution gargle
d. Iodized salt
96.
97.
98. Situation 14- Maricar, a staff nurse assigned in the medical ward reports during the morning shift. All
the assigned to Maricar have ongoing intravenous therapy. To ensure safe and quality nursing care,
Maricar implements policies, procedures and guidelines set by the hospital regarding intravenous
therapy.
99. At 1000 H, the attending physician prescribed for a client who is on her 2 nd day post hemicolectomy,
Dextrose in Water 1000ml with 20 mEq Potassium Chloride to run for 8 hours.If the intravenous therapy
was started at 1100 H, how many ml of intravenous solution will Maricar anticipate to have been infused
when she hands off her client to the incoming shift nurse at 1400H.
a. 375ml
c. 300ml
b. 350ml
d. 320ml
100.
After incorporating 20 mEq Potassium Chloride into the Dextrose 5% in Water 1000ml bag, the nurse
AVOIDS doing which of the following nursing intervention?
a. Shake the IV bag
b. Place calibration label on the IV bag
c. With a pen marker , label the IV bag with the incorporated drug
d. Check for color changes in the IV bag
101.
When Maricar checked on the intravenous infusion of one of her clients, she noted a label attached to
the intravenous tubing with the date 6/23/12. She understands that the intravenous set will be changed on
a. 6/25/12
c. 6/27/12
b. 6/24/12
d. 6/26/12
102.
The client with ongoing intravenous infusion of Dextrose 5% lactated Ringers solution rings the call
bell and when the nurse approached her , she pointed to her intravenous (IV) site. When the nurse
assessed the IV site, she noted that phlebitis has developed. The nurse does the following nursing
intervention EXCEPT:
a. Restart an IV line in a proximal portion of the same arm
b. Reinstruct the clients what not to do while with IV infusion
c. Applies cold moist compress over the IV site
d. Elevate the affected arm on a pillow
103.
An elderly client with an ongoing of dextrose 5% in NaCI 0.9% 1000 ml hung at 1545 H was assessed
to be slightly dyspneic, chilling and with increased pulse rate. The IV bag has 400ml remaining and it was
1680 H. the nurse should take which IMMEDIATE nursing action?
a. Refer to the attending physician STAT
c. Slow down the IV infusion
b. Remove the IV cannula
d. Put the client in a sitting position
e.
f. Situation 15- Andrew, a 12 year old boy with type I diabetes mellitus, is admitted the medical
ward from the Intensive Care unit after having recovered from an episode of diabetic
ketocidosis. Andrew has been diagnosed with Type I diabetes since he was 6 years old.
History showed that during the past 2 months. Andrew missed some of his insulin. Injections
as he got himself engrossed playing tennis.
104.
The admitting nurses noted that Andrew is underweight and short of stature. The nurse considers
which of the following reasons BEST explain Andrews retarded growth?
a. Large amounts of protein and fat are used for energy
b. Occurrence of electrolyte imbalance leading to dehydration
c. Increased breakdown of fats for cell utilization
d. Inability to use glucose as a source of energy
105.
When the nurse plants for Andrews insulin injection sites, which of the following sites will the nurse
NOT include in her plan?
a. Upper outer part of dominant arm
b. Outer part of the thighs
c. Four inches above the knee of both thigh
d. Abdominal subcutaneous tissue just below the waist
106.
To ensure that an injection site will not be repeated when nurses administer insulin on Andrew, which
of the following nurses action would most effective?
a. Have nurse record in the childs chart the injection site
31

b. Every shift, verbally endorse to the receiving nurse the injection site
c. Instruct the patient to tell the nurse, the site use during the previous injection
d. Mark with a ballpen the injection site previously used
107.
In the teaching plan being prepared by the nurse for Andrew , which of the following strategies would
be most relevant for Andrew to a void overuse of an injection site for insulin self administration?
a. On a teaching doll, injection sites are marked with green colored pins. After injection , the pins are
replaced with red colored pins to indicate site has been used
b. A chart is prepared illustrating body parts where injection sites are determined for a month. After
injection, site is marked with the date and time of injection
c. On a record book, injection sites are enumerated daily for one month. Every after injection, date and
time are recorded across the used of injection site
d. Paper doll is constructed. Injection sites are determined for a week . Injection sites are marked on
the paper doll. Site is crossed out.
e.
f.
g.
h.
i.
j.
108.
Which of the following statements of Andrew will the nurse consider as an indication that Andrew
is ready to self administers his insulin?
a. Will you allow me to do it now?
c. When I go home, I will do it myself
b. Let me hold the syringe for you
d. Are you sure I can do it myself?
e.
f. Situation 16- The medical and surgical unit where you work just hired 3 nurses to
augment the present nursing human resources. The following questions apply.
109.
You are assigned one new nurse to work with during the shift. An admission from the Post
Anesthesia Care Unit (PACU) of a post thoracotomy with wedge resection with a chest tube came in
and you assigned the nurse to do initial assessment. Which assessment observed will you report to the
surgeon right away?
a. 80ml of dark red output from the drainage bottle
b. Intermittent bubbling in the suction control
c. Intact and dry dressings
d. The drainage system is hanged at the bed side below the clients chest
110.
You put the client in Fowlers position and explain the rationale before the client and significant
other the benefit of this position. If you were the nurse, which would be the BEST reason for the
Fowlers position?
a. Relaxes the sterna muscles and enhances breathing
b. Promotes deep breathing and reduces pain during inspiration
c. Reduces pressure on the diaphragm and permits lung expansion
d. Increases pressure on the diaphragm and allows optimal expulsion of secretion
111.
The new nurse reads the doctors order Maintain patent chest tube and close drainage milk
tubing pm.the APPROPRIATE nursing action is:
a. Pinch the tubing alternately towards the drainage chamber if there is visible fibrin or clots
b. Clamp the tubing every time the client coughs
c. Milk feeding
d. Empty the drainage tube prn.
112.
The new nurse encouraged the client to assume a comfortable position while maintaining body
alignment despite the presence of the drainage system. While the patient was looking for a more
comfortable position, the tubing was accidentally disconnected. The INITIAL and APPROPRIATE action
of the nurse is to:
a. Clamp the tubing at once
b. Place the open end of the tubing in a sterile water
c. Pull out the tubing and apply an air tight dressing on the site
d. Immediately reconnect the tube
113.
The senior nurse was emphasizing to the new nurse that intermittent bubbling of the water seal
chamber is normal but should continuous bubbling be observed, this can indicate
a. Presence of air leak
b. That pressure is equal to the water seal
c. No more air is leaking into the pleural cavity
d. Negative pressure in the medlastinal cavity
e.
f. Situation 17: Nurse Linda is assigned to the pediatric surgical unit to take care of JV and
Billy.
114.
JV, 18 months, was admitted for repair of hypospadias. During assessment. Which of the
following will Linda expect to observe?
a. Absence of urethral meatus
32

b. Termination of the urethra is in the ventral surface of the penis


c. Defect of the urethra on the dorsal surface of the penile shaft
d. Penis has 2 urethral openings located dorsally and ventrally
115.
Surgery is the treatment of choice for JV. The nurse understands taht the best time for surgery is
before the child:
a. Is weaned from diapers
c. Goes to school
b. Is toilet trained
d. Walks
116.
Linda prepares a nursing care plan for JV. Postoperatively, which of the following is a PRIORITY
nursing diagnosis?
a. Risk of infection
c. Potential malnutrition
b. Alternation of fecal elimination
d. Altered body image
117.
Billy, 1 year old, was admitted to the unit from the recovery room post cheiloplasty. Linda would
place Billy in which of the following positions?
a. Lateral
b. Fowlers
c. Supine
d. Prone
118.
119.
120.
121.
When billy fully recovered from anestnesia , the doctor ordered clear liquids as tolerated. Which
of the following is the APPROPRIATE action of the nurse/
a. Allow infant to sip from a cup
b. Use spoon and feed slowly and gently
c. Administer liquids through a medicine dropper
d. Bottle feed the infant
122.
123.
Situation 18- Nurse Carmina is preparing like, a 28 year old newly for surgery for a repair a
multiple trauma from a car accident like is in severe pail and comforted by this wife and
significant others.
124.
There exist a hierarchy who should sign the consent to be legally valid if the client is not
competent. Rank the following next of kin who shall sign the consent for likes surgery
125.
1. Grandparents from paternal or maternal side
126.
2. Adult competent children
127.
3. Brother or sister
128.
4. Legitimate spouse
129.
5. Guardian whether appointed by court or not
a. 4,5,3,2 and 1
c. 4,2,1,3 and 5
b. 4,3,1,2 and 5
d. 5,3,4,2 and 1
130.
Legally, nurse Carmina shall assume which role during the signing of the consent?
a. Advocate
c. Interpreter
b. Witness
d. Counsellor
131.
Like underwent exploratory laparotomy for multiple organ injuries in his abdomen. Which doctrine
is applied when the surgeon is held liable when there is an incorrect surgical count?
a. Res Ipsa Loquitor
c. Doctrine of Vicarious Liability
b. Captain of the Ship
d. Doctrine of Independent Contractor
132.
During the surgery, the client was profusely bleeding that prompted the surgeon to verbally order
Transfuse all available blood. Which of the following options would the nurse take so that she will not
be liable if blood transfusion complications occur?
a. Document as ordered and have the surgeon sign as soon as feasible
b. Transfuse the blood with the anaesthesiologist
c. Send the blood for proper cross matching and transfuse immediately after
d. Leave the anaesthesiologist to follow the order
133.
The surgeon is such in hurry to closebecause of the deterioting condition of the client. The
perioperative nurses cannot account for an operating sponge (OS). Which is the MOST appropriate
action of the scrub nurse at this point?
a. Hands the suture for closing and tell the surgeon that one OS cannot be accounted for
b. The scrub nurse asks the circulating nurse to recheck the sponges one or more time
c. The scrub nurse informs the surgeon that one OS cannot be accounted for
d. Obligingly , the scrub nurse hands the suture to close and continue to locate the missing OS
134.
135.
Situation 19- Nurse Alpha is caring for Edwin 40 year old 3rd day post bowel resection,
NPO with D 5 LR IV 1000ml at 125 ml/hr. Laboratory findings show a haemoglobin level of 8 g/dl
and hematocrit of 30%
136.
137.
During the physician rounds , Dr. Grande made the following orders
33

- Gentamycin 80 mg IV piggy back in 50 ml D 5 water over 30 minutes


- Ranitidine 50 mg IV in 50 ml D5 water piggly tack in 30 minutes
- Packed red blood cells (RBC) 25oml to run for 3 hours
138.
How many millimeters should Nurse Alpha document as the total intake for the 8 hour
shift?
a. 1000ml
c. 1350ml
b. 350ml
d. 1300ml
139.
While reading Edwins chart, you read the laboratory findings as
- Serum potassium 2.2h mEq/L
- Sodium 129 mEq/L
- Calcium 7.5 mg/L
140.
The nurse would anticipate / prepare which of the following IV solutions to be
prescribed?
a. Sodium chloride 0.45%
b. Dextrose 5% in water
c. Dextrose 5% in Lactated Ringers solution
d. Nomosol
141.
142.
143.
144.
145.
146.
147.
148.
149.
Nurse Alpha continued to monitor Edwin who has an ongoing IV and Packed red blood cells
(PRBC) transfusion. The client complains of headache, backache and the temperature began to spike.
Rank the action of the nurse according to PRIORITY:
150.
1. Refer to the attending physician
151.
2. Assess the client
152.
3. Close the roller clamp of the PRBC
153.
4. Keep the vein open with NSS
5. Document observation and intervention
a. 2,3,4,1 and 5
b. 3,2,1,4 and 5
c. 3,4,2,1 and 5
d. 1,2,3,4 and 5
154.
Nurse Alpha identifies risk for wound complications. in case of wound evisceration , the
IMMEDIATE action of the nurse is to
a. Instruct the client to stay quiet in bed as you call for help
b. Apply clean abdominal binder and place the pillow on top of the wound
c. Cover the wound with sterile gauze wet with sterile NSS
d. Call for the surgeon stat
155.
Edwin has been on NPO since he was operated and asks the nurse when he can have food.
Nurse Alpha most APPROPRIATE response is
a. The dietetian will make their rounds in a while to assess you and other postoperative clients
b. The surgeon will make their rounds to assess your readiness to take in your preferred diet
c. Clear soup will be serves as soon as you have bowel sounds
d. You can have sips of water for the mean time
156.
157.
Situation 20- Susan, 40 years old and weights 180lbs, underwent clolecystectomy for
cholelitheasis and gall stones.
158.
159.
Susan complained of incisional wound pain as she recovered from anesthesia . she has n order
of Demerol 75 mg every four hours round the clock for pain. The nurses CORRECT intervention is:
a. Inject Demerol 75 mg as ordered for pain
b. Instruct the client to apply pressure over the operative site
c. Encourage Susan to do deep breathing
160.
Because of the fear of wound gaping and pain. Susan was observed suppressing her cough
reflex. The MOST appropriate nursing intervention to minimize pain in every coughing episode is to:
a. Splint the operative site with wide plaster
b. Advise the client to turn to sides every 30 minutes
c. Give sedation round the clock to minimize coughing and pain
d. Instruct the client to splint the incision wound with pillow
161.
Which psychological nursing intervention can you offer to minimize postoperative pain?
a. Stay with the client and offer to comb her hair and put some make up in front of the mirror
34

b. Allow client to converse with other clients regarding their experience in coping post operative
pain
c. Instruct client and significant others to avoid talking about pain within the hearing distance of the
client
d. Restrict visitors that can aggravate noise and destruction of rest
162.
One cause of post operative pain is incision wound contamination. Hence, nurses and other
members of the health team are expected to do cheap and effective infection prevention measures
such as
a. No touch technique
b. Change gloves in between patient
c. Hand washing between client
d. Use alcohol swab if hand washing is not permissible
163. Post operative pain keeps surgical clients in bd. Most surgical clients like Susan are encouraged to
ambulate:
a. As soon as indicated
b. Within 6 to 8 hours surgery
c. Between 10 to 12 hours after surgery
d. On the second postoperative day
164.
165.
166.
167.
168.

169. A1 PASSERS TRAINING, RESEARCH, REVIEW AND DEVELOPMENT COMPANY


170.
PRE-BOARD EXAMINATION
171.
172.
NURSING PRACTICE IV-Care of Clients with Physiologic and Psychosocial Alteration
173.
174.
175.
GENERAL INSTRUCTIONS:
176.
1. This test booklet contains 100 test questions.
177.
2. Read INSTRUCTIONS TO EXAMINEES printed on your answer sheet.
178.
3. Shade only one (1) box for each question on your answer sheets. Two or more boxes shaded will
invalidate your answer.
179.
4. AVOID ERASURES.
180.
181.
INSTRUCTIONS:
182. 1. Detach one (1) answer sheet from the bottom of your Examinee ID/Answer Sheet Set.
183. 2. Write the subject title Nursing Practice I on the box provided.
184. 3. Shade Set Box A on your answer sheet if your test booklet is Set A; Set Box B if your test booklet is
Set B.
185.
_________________________________________________________________________________
_________
186.
187.
Situation 1-Juanchito, 5 yrs. old, has idiopathic nephritic syndrome. He has generalized
edema with a puffy face, distended abdomen and edematous legs. Blood pressure is normal.
Blood tests show hypoalbuminemia.
1. The nurse is aware that generalized edema is due to hypoalbuminemia which lead primarily to which of
the following?
a. Increased secretion of antidiuretic hormone
b. Reduced intravascular volume
c. Decreased plasma osmotic pressure
d. Stimulation of the rennin anglotensin system
2. The nurse closely monitors the urine output of the patient. Which of the following characteristics of a
urine sample will the nurse expect?
a. Fruity odor
c. Urine is frothy
b. Increase amount
d. Blood in urine
3. The attending physician of Juanchito prescribed renal biopsy. When the nurse plans for the nursing
care of Juanchito after the biopsy, which of the following will be a PRIORITY intervention to prevent
bleeding?
a. Observe for abdominal pain and
c. Place on complete bed rest
tenderness
d. Closely watch urine output
b. Monitor vital signs
4. Cortecostiroid therapy was prescribed. Which of the following is the MOST relevant nursing intervention
to address complications of the therapy?

35

a. Weigh daily to monitor fluid balance


c. Maintain on a salt restricted diet
b. Closely monitor for changes in body
d. Offer small frequent meals
temperature
5. When the nurse prepares her health instruction for the mother of Juanchito, which of the following side
effects of the drug will the nurse include in her plan?
6. 1. Diuresis
7. 2. Hirsutism
8. 3. Abdominal distention
9. 4. Loss of appetite
10. 5. Rounding of the face
a. 3,4,5
c. 1,2,5
b. 2,3,5
d. 1,2,3
e.
f. Situation 2- as ED nurse you should always be ready to any kind of emergency situation
11. Peter , a 30 year-old factory worker, is brought to the emergency department with multiple lacerations
and tissue avulsion of the right hand after catching the hand in a produce conveyor belt. When asked
about the tetanus immunization , he says, Ive never had any vaccinations, You will anticipate
administration of:
a. Tetanus toxoid
b. Immunoglobulin and tetanux-diphtheria toxoid
c. Immunoglobulin
d. Immunoglobulin, tetanus diphtheria toxoid, and pertussis vaccine
12. Paul is brought to the ED by his co worker after suffering from amputation of a left middle finger. As his
nurse which of the following should you AVOID to do?
a. Placing the wrapped finger in a plastic bag
b. Cleansing the stump area with normal saline
c. Wrapping the amputated finger in sterile gauze moistened with saline
d. Placing the amputated finger directly on ice
13. Josh, a 7 year old child is brought to the ED by his mother complaining of arm pain after falling off a
swing at alcohol. After assessing Joshs pain on a scale of 1-10, what will be your PRIORITY activity?
a. Assume that Josh is too young to verbalize how much pain he is experiencing
b. Obtain further information about his pain using age appropriate tool
c. Give Josh a narcotic pain medication
d. Ask his to wait in the waiting room to find out if Josh gets better without his mother
14. During the primary assessment of Bernie, who has sustained multiple trauma, you observe taht his right
pedal pulses are absent and the leg is swollen. Your PRIORITY action is to
a. Initiate isotonic fluid infusion through two large bore IV lines
b. Finish the airway, breathing, circulation , disability survey
c. Send blood to the laboratory for a complete blood count (CBC)
d. Assess further for a cause of the decreased circulation
15. Anthony, 20 years old college student , unconscious , is admitted to the ED about 45 minutes after
ingesting approximately 30 diazepam (Valium) tablets. The physician prescribes gastric lavage. As a
nurse , your FIRST action when implementing the order is to
a. Insert large bore nasogastric tube
b. Position Anthony on his side
c. Assist the physician to intubate Anthony
d. Prepare a 50 ml syringe with saline
e.
f. Situation 3- Alicia a newly hired staff nurse is assigned to a patient diagnosed with
Diabetic Ketoacidosis with Urosepsis in the Intensive Care Unit
g.
16. Physician prescribed that Capillary blood glucose (CBG) be monitored every 1 hour. Alicia missed to
check the patients capillary blood glucose at 1000H. Which of the following decisions of Alicia
regarding documentation of hourly CBG is MOST appropriate?
a. Sign the space of the data to indicate CBG reading not done
b. State reason for not having taken the CBG reading
c. Document per hospital protocol
d. Leave the space that corresponds to 1000H blank
17. Alicia was taught on electronic record keeping and became aware that the following are the benefits of
electronic record keeping EXCEPT:
a. Prevents tampering with a patients record
b. Password secures accessing the system
c. Eliminates problem of illegible handwriting
d. Increases time required for documenting
18. To maintain confidentiality of patients record. Alicia recommended which of the following practices
needs improvement?
36

a. Screen visible to passersby


c. Access code are not shared by the
b. Computer log off automatically when
nurses
not in use
d. Printouts are retrieved immediately
19. Alicia looked for a manual which could facilitate her orientation regarding documentation the manual is
not available. This observation reflects management inadequacy in which of the following?
a. Controlling
b. Planning
c. Directing
d. Organizing
20. Standards of documentation require that a system is in place regarding corrective actions related to
errors in charting. Which of the following observations in the patients record indicates taht the standard
is implementation?
a. Updated information with notation marked as late entry
b. Correction of the erroneous entry written above
c. Corrected information written in the next available space
d. A straight line drawn across a phrase in the nurses notes, dated physicians initialled
21.
22.
23.
24.
25.
26.
27.
28.
29. Situation 4- Diomedes, a 56 year old farmer, was admitted in the Emergency Department (ED)
because of hematemesis accompanied by hematochezia. Diomedes is an alcoholic and is under
treatment for cirrhosis of the liver. His abdomen is enlarged and his lower extremities are
edematous. Admitting physicians initial diagnosis is ruptured esophageal varices.
30. Assessment reveals signs and symptoms of early compensatory hemorrhagic shock. If you were the
nurse who admitted. Diomedes, which of the following will you consider as the compensatory
mechanism responsible for the increased heart rate and respiratory rate?
a. Stimulation of the sympathetic nervous system
b. Increase in size of the vascular bed to peripheral vasodilation
c. Rennin angiotensin response
d. Release of adrenocorticotropic hormone from the hypothalamus
31. To restore hemodynamic stability on the client, which of the following will the nurse expect to be done
FIRST?
a. Insertion of central arterial and venous catheters
b. Endoscopic ligation of rupture varices
c. Blood transfusion of blood replacement
d. Administration of vasoactive and inotropic drugs
32. Hemodynamics measurement revealed stable vital signs and increased cardiac output. The physician
ordered treatment of the esophageal varices. Which of the following procedures will the nurse expect to
be done?
a. Upper endoscopy
c. Exploratory laparotomy
b. Intrahepatic portal systemic shunt
d. Coagulation therapy
33. In the intensive care unit nursing orders required all nurses to assess regularly for early manifestations
of portal systemic encephalopathy. Which of the following will the nurse note during her observation?
a. Occurrence of esterexis
b. Development of disorientation and incoherence
c. Signs and symptoms of increased intracranial pressure
d. Presence of papilledema
34. Serum ammonia level of the client remained to be elevated. The following may be considered by the
nurse to be true regarding this observation except?
a. Ammonia is formed as proteins and amino acids are broken down by intestinal bacteria
b. Ammonia accumulates in the blood due to inability of the kidney to excrete ammonia
c. Due to bleeding, blood in the intestinal tract is digested as protein, thereby increasing serum
ammonia
d. Since liver function is destroyed , ammonia can no longer to be converted to less toxic form.
35.
36. Situation 5- The declining level of patient satisfaction related to nursing service per survey
results as well as increased incidences of hospital acquired infection during the past 6 months
caused the nursing service division to push the nursing units to explore quality improvement
projects.
37. The Intensive Care Unit (ICU) Quality Improvement team decided to gather data to determine probable
causes of central line infection among the ICU patients, if you were the member of the quality
37

improvement team, which of the following data will you consider as MOST appropriate to yield the
most probable cause of central line infection?
a. Nurses notes on hourly assessment of the sites of central line
b. Performed central line care interventions as observed
c. Daily every shift report of central line care measures from bedside nurses
d. Incidence of central line infection as reported by infection control nurse
38. The highest incidence of fall among the hospitalized patient is in the medical unit. The medical units
quality improvement team has identified the probable causes of the incidences of fall among their
hospitalized patients. With the data analyzed and findings organized . which of the following should the
quality improvement team do first?
a. Implement prevention measures identified to be effective
b. Propose a list of nursing actions intended to identify fall risks and preventive measures
c. Do a pilot study of the fall prevention measures to a small group of patients
d. Brainstorm for a plan for an appropriate action for change
39. Another group of quality improvement team in the ICU conducted a project of ventilator associated
pneumonia incidences among ICU patients. If you are a member of this team. Which of the following
measures will you consider as the MOST appropriate to be implemented in collaboration with the
respiratory therapist?
a. Perform regularly assessment of the clients readiness to be extubated
b. Consider orotracheal as a preferred route of endotrachealintubation
c. Maintain head elevation at 30-45 degress
d. Suction endotracheal tube as prescribed in the manual of procedures
40.
41.
42.
43. Noise level in the ICU has always been a complaint in the patient satisfaction survey. Which of the
following tools can be recommended to the quality improvement team as most appropriate to determine
level of noise in the ICU?
a. Questionnaire with clients and patients as respondents
b. Observation checklist
c. Measurement device
d. Interview schedule form with nurses , clients and relatives as interviewees
44. During a group discussion , probable factors responsible for urinary tract infection incidences among
hospitalized clients in the medical unit were being explored , which of the following will you consider as
the group of data would LEAST helpful?
45. 1. Diameter and length of Foley catheter
46. 2. Length of time Foley catheter has been kept indwelling
47. 3. Age and sex of client
48. 4. Daily physical activities of the client
49. 5. Relevant data regarding need for continuing indwelling catheter
a. 1,2,4
c. 1,2,3
b. 3,4,5
d. 2,3,4
e.
f. Situation 8- Kiko, 8 years old, has two chest tubes connected to a disposable water seled
drainage system because of chest injuries from a vehicular accident.
50. The nurse observed that the drainage from the chest tubes have not increased from the previos shift
report. Which of the following is the PRIORITY action of the nurse?
a. Check the chest tube for kinks
c. Document observation in the chart
b. Assess for breath sounds
d. Change position of the patient
51. Frequent assessment of the closed drainage system is important to ensure appropriate functioning. The
nurse observes that water level fluctuates with respiratory effort. The nurse considers this as a sign of
a. Trapped air
c. Patent tubes
b. An inefficient system
d. Air leaks
52. The nurse works with a nursing aide. Which of the following is a CORRECT action of the nurse? The
nurse directed the nursing aide to:
a. Always check that clamp is available at the bedside
b. Observe regularly the amount and color of drainage from chest tubes
c. Report signs of patients discomforts at the site of the chest tubes
d. Turn the patient regularly and maintain connections of the tubes
53. While the nurse was turning the patient during bed bath. One of the chest tubes was pulled out from its
site. Which of the following will the nurse do FIRST?
a. Reinsert the chest tube
b. Disconnect chest tube from drainage system
c. Cover wound site occlusively
d. Clamp the chest tube
38

54. To determine if chest tubes are in place and pneumothorax is corrected. Which of the following will the
nurse expect the physician to order?
a. Tidal volume measurement
c. Chest radiograph
b. Arterial blood gas analysis
d. Thoracentesis
55. If you are a member of a research team, which of the following will you consider as the research study
independent variable when you formulate the research problem?
a. Incidence of infection
b. Relationship of endotracheal suctioning and incidence of infection
c. Dependence of clients on mechanical ventilation
d. Frequency of endotracheal suctioning
56. Data regarding the study dependent variable will be collected by the research team through which of the
following?
a. Self report technique
c. Projective techniques
b. In vitro measures
d. Available data in the patients chart
57. The research team is fully aware that measurement of variables is a very important consideration in
obtaining quality data in the study. Which of the following statements will you accept as TRUE?
a. Reliability quality of an instrument is independent of its validity
b. An instrument can be valid without being reliable
c. A measuring device which is unreliable can be valid
d. High reliability of an instrument provides no evidence of its validity
58. Taking into consideration of the content of the written informed consent. Which of the following reflects the
research teams recognition of the participants right to privacy?
a. Right to withdraw and withhold
c. Voluntary consent
information
d. Potential benefits and risks
b. Confidentiality pledge
59.
60.
61. The research team decided to conduct the study for 3 months. Utilizing the prospective approach.
Which of the following will the research appropriately do?
a. Participants will be assigned to the experimental and control group and incidence of infection in
two groups will be compared
b. During the period of 3 months, those ventilator assisted patients who developed infection will be
included in the study
c. On the last day of the 3rd month charts of participants be reviewed and data collected regarding
frequency of endotracheal suction and incidence of infection
d. From day one to the last of the 3rd month, data regarding frequency of endotracheal suctioning
and incidence of infection will be collected
62.
63. Situation 8- A 69 year old female patient was admitted in the Emergency Department (ER) via an
ambulance from a nearby restaurant. Patient was drowsy and was unable to identify herself.
Blood pressure was 150/90. Tachycardia and respiration shallow with a rate of 30/minute.
Companion of the patient claimed that while laughing, the patient got choked with a piece of
pork meat and was unable to breathe for quite some time until someday successfully got it out
of her throat.
64. Physicians admitting diagnosis is acute respiratory acidosis. If you were the ED nurse who admitted the
client. Which of the following will you expect?
a. A normal pH and a PaCO2 greater than 45 mmHg
b. A pH more than 7.35 and a PaCO2 lower than 45 mmHg
c. A pH less than 7.35 and a PaCO2 greater than 45 mmHg
d. A pH less than 7.35 and a PaCO2 of 45mmHg
65. The initial arterial blood gases results of the client revealed a normal bicarbonate level which of the
following statements will the nurse accepts as TRUE that would explain this specific finding?
a. Serum bicarbonate will remain unchanged only the excretion rate of acids
b. The kidneys can modify only the excretion rate of acids
c. The compensatory response of the kidneys occurs only over hours to days
d. A change in serum bicarbonate is noted only in cases of respiratory acidosis
66. You are understands that the alternation in the mental status of the client is primarily due to the acute
effects of which of the following?
a. Hyperventilation
c. Tachypnea
b. Hypercapnia
d. Hypoxemia
67. Related to temporary airway obstruction, the admitting nurse identified. Impaired gas exchange; as a
priority nursing diagnosis. Which of the following interventions will you consider to be the MOST
appropriate to determine if normal gas exchange has been regained?
a. Administer prescribed bronchodilator
b. Maintain on oxygen inhalation as ordered
c. Place on Fowlers position as tolerated
d. Monitor arterial blood gases redrawn every 2 hours
39

68. The client is closely observed for signs of dysrrhythmia. If you were the nurse at the bedside, which of
the following waves in the cardiac monitor will you consider as the wave that represents the contraction
of the ventricles?
a. QT interval
c. PR interval
b. QRS complex
d. ST segment
e.
f. Situation 9- Lito, 35 years old is HIV positive. Aside from fatigue. Lito has no other
complaints
69. The result of the Enzyme-linked immunosolvent Assay Test (ELISA) was positive. The nurse
understands that this is attest to determine the presence of
a. HIV antibody
c. HIV antigen antibody response
b. Actively replicating HIV
d. Increased CD4 cell count
70. Which of the following is NOT a primarily tool for AIDS precention?
a. Education
c. Immunization
b. Counselling
d. Behavior modification
71. Which of the following is an APPROPRIATE advice of the nurse for Lito?
a. Do not engage in unprotective sexual activity
b. Inform all health care personnel
c. Inform all sexual partners of his health status
d. Do not become pregnant
72. From the list of nursing diagnosis prepared by the nurse for Lito. Which of the following is a PRIORITY
nursing diagnosis?
a. Risk for impaired skin integrity
b. Risk for injury
c. Ineffective coping
d. Ineffective sexuality pattern
73.
74.
75. When the nurse interacts with the clients, which of the following precautionary measures should she observe
to prevent contamination?
a. Avoid touching body fluids
c. Maintain a 3 feet distance from the
b. Wear gloves
patient
d. Wear mask and gown
e.
f. Situation 10- Leon has been diagnosed with End Stage renal Disease. The physician
prescribed dietary teaching and outpatient hemodialysis three times a week
76. Leon asks the nurse to tell him the purpose of the treatment. which of the following is the MOST appropriate
response of the nurse/
a. Hemodialysis removes excess fluids and waste products and restores electrolyte balance
b. Hemodialysis uses the principles of diffusion and ultrafiltration to remove electrolytes
c. Blood is pumped through a semipermeable capillary in a hemodialyzer
d. Hemodialysis is one of several renal replacement therapy
77. An intravenous fistula has been created. Postoperatively, which of the following will the nurse include as a
PRIORITY nursing intervention to promote circulation?
a. Auscultate for bruit every 4 hours
c. Observe finger tips for cyanosis
b. Elevate the affected arm
d. Keep dressing intact
78. Which of the following pre dialysis care is done by the nurse to be able to determine effectiveness of
treatment with regards to excess fluid volume?
a. Assess integumentary status
c. Have patient empty bladder prior to
b. Assess vascular site
treatment
d. Record weight and vital signs
79. Nutrition therapy of Leon includes control of protein. Dietary prescription states that leon is allowed 0.8 gram
of protein per kg per day. If Leon weighs 120lbs, how much is his daily protein allowance?
a. 57.9 g
c. 81.7g
b. 43.6g
d. 96g
80. Leon claims he loves to eat raisins. The nurse instructs the patient to avoid this food because it is rich in
which of the following?
a. Sodium
c. Magnesium
b. Potassium
d. Phosphorous
e.
f. Situation 11- Arnel , 58 years old, post total thyrodectomy with modified neck dissection due
to papillary carcinoma of the thyroid gland with lymph node metastasis was admitted for
radioactive iodine therapy.
81. Prior to admission, the client underwent a scan with a test amount of radioactive iodine. If you were the
nurse who admitted the client, which of the following will you consider as the reason for this intervention?
a. To determine the existence of known distant metastatic tumor
b. To measure size of remaining thyroid tissue
c. To explore the operative site for baseline data
40

d. To mark the site where the radioactive iodine will be administered


82. In the nursing care plan prepared for the client, which of the following nursing interventions is LEAST
relevant to ensure a safe environment once treatment has started?
a. Utilize preferably only disposable items for patients personal use
b. Provide hand sanitizers in the corridor outside the clients room
c. Dispose appropriately garbage bags marked radioactive
d. Have all frequently handled items in the room covered with absorbent material
83. As the nurse assigned to the client, you understand that after radioactive iodine has been administered,
excess iodine not absorbed by the thyroid tissue, will leave the body PRIMARILY through which of the
following?
a. Sweat
c. Feces
b. Urine
d. Saliva
84. To ensure effectiveness of the radioactive iodine therapy, you expect the physician will prescribe low iodine
diet during which of the following?
a. Day of administration of the radioactive iodine until day of discharge
b. Two weeks before, during and until 2 days after the treatment
c. The day before the scan until the first day after the treatment
d. Upon admission in the hospital until a week after the treatment
85. When planning discharge, which of the following instructions will you consider for reduction of radioactive
exposure to others?
86. 1. Use private toilet facilities and flush 2-3 times after use
87. 2. Wash eating utensils separately from others
88. 3. Drink normal intake of fluids
89. 4. Bathe daily and wash hands frequently
90. 5. Stay in isolation at home two weeks after the treatment
a. All except 3
c. 2,4 and 5
b. 2,3 and 5
d. 1,2 and 4
e.
f.
g.
h. Situation 12- Manuel, 4 years old is positive for Bacterial Meningitis
91. From the history obtained from the mother, which of the following could be the possible method by
which the infection was transmitted to the patient?
a. Drinking water in the community was contaminated
b. Contact with respiratory secretions of an infected person
c. Hand of caregiver was contaminated with fecal discharges
d. Eating utensils of the child were contaminated
92. The physician prescribed lumbar tap. When the nurse reads the laboratory results, which of the
following reflects positive results indicative of Bacterial meningitis?
a. Decreased white blood cells, decreased proteins, high glucose
b. Normal white blood cells count , increased proteins, high glucose
c. Increased white blood cells, increased proteins, low glucose
d. Increased white blood cells, decreased proteins, low glucose
93. Assessment findings reveal positive Brudzinskis sign. When the nurse flexed the Childs neck forward.
Which of the following behavior indicated a positive Brudzinskis sign?
a. Hip flexed and knee extended
c. Leg extended with resistance
b. Knee extended and ankle flexed
d. Hip, knee and ankle flexed
94. In the nursing care plan prepared by the nurse, Pain related to meningeal irritation: is a priority nursing
diagnosis. Which of the following should the nurse avoid to do to prevent pain when positioning the
patient?
a. Extend leg
c. Hyperextend the neck
b. Flex the neck forward
d. Flex the hip
95. Ineffective tissue perfusion related to increased intracranial pressure is another nursing diagnosis
formulated by the nurse. Which of the following assessment data specific to eye changes would t he
nurse interpret as normal intracranial pressure?
a. Positive sunset eye signs
c. Positive nystagmus
b. Positive strabismus
d. Positive dolls eye reflex
e.
f. Situation 13- You are assigned in the Nephrology Ward. One of your patients is Carlos
with an admitting impression of right renal calculi. Based on his history, Carlos was
brought to the ED by his friend when he experienced severe excruciating right flunk pain,
nausea and vomiting. This was relieved when the doctor gave him Buscopan. The doctor
ordered several diagnostic works up: Kidney urinary Bladder and intravenous Pyegraphy
(KUB-IVP) and ultrasound. Blood chemistry and 24 hour urine collection to measure
calcium, uric acid, creatinine, sodium, pH and total volume were, likewise ordered.
96. As a nurse of Carlos who is for KUB-IVP, which of the following will you include in your teaching plan?
a. 1. What is an intravenous pyelography (IVP)

41

b. 2. How should I prepare for this procedure?


c. 3. How is the procedure performed?
d. 4. What will experience during and after the procedure?
e. 3 and 4
g. 1 and 2
f. All of the above
h. All except 3
97. One of the physicians orders in 24 hour urine test. What instruction will you give Carlos if the collection
will start tomorrow at 7:00 AM (Day 1) and end at 7:00 Am the following day (day 2)?
a. Discard your first urine sample at 7:00 AM tomorrow then start urine collection until 7:00 AM day
2
b. Start urine collection at exactly 7:00 AM tomorrow up to day 2 but discard the last urine at 7:00
AM of day 2
c. Discard your first urine sample tomorrow at 7:00AM and your last urine sample collection on Day
2 at 7:00AM and collect all urine samples between those times
d. Start urine sample collection at exactly 7:00 AM tomorrow
98. The diagnostic examination confirmed the presence of renal calculi. A dietary medication and family
history of renal stone was part of the assessment that was done in order to
a. Avoid taking drugs that could have contributed to stone formation
b. Identify the factors predisposing Carlos to formation of stone
c. Prescribe the type of diet that is needed to prevent recurrence
d. Identify what type of stone was formed
99. To facilitate spontaneous passage of stone and dilute the urine , the following can be advised for
Carlos EXCEPT:
a. Limit fluid intake to 1.5 liters per day to avoid overload
b. Examine all urine output for presence of stone
c. Promote sufficient fluid intake to maintain a urine output of 3-4 liters/day
d. Record intake and output and daily weight to assess fluid status and renal function
100.
101.
102.
103.
Stone assay was done following its spontaneous passage. The result revealed uric acid stone.
Prior to his discharge, you made a teaching plan for Carlos on how he can prevent kidney stone
formation. The following were included in your plan EXCEPT:
a. Take Allopurinol (Zyloprim) that is prescribed by the doctor to reduce serum uric acid levels and
urinary uric acid excretion
b. Avoid food high risk like shellfish, anchovies, mushroom and organ meal
c. During the waking hours drink fluids every 1-2 hours and at bedtime take 2 glasses of water to
prevent urine from being too concentrated
d. Encourage activities leading to sudden increases in temperature to facilitate excretion of uric acid
through sweating
104.
105.
Situation 14-EMERGENCY-Triage to facilitate care of clients in the emergency room,
various management strategies have been devised to address the survival needs of patients. As
an ER nurse you should be equipped with knowledge, skills and attitude to cope with
unexpected problems:
106.
You are assigned as the triage nurse in the ER . Four patients injured in a vehicular accident
were brought to the Emergency Room at the same time. To whom will you assign the HIGHEST priority
a. Marissa , with macillofacial injury and gurgling respiration
b. Rebecca , with severe head injury but with no perceptible blood pressure
c. Rose, with lumbar spinal cord injury with lower extremity paralysis
d. Charlene, 8 months pregnant with premature labor contraction
107.
Remund, was sideswiped by a motor cycle while he was waiting for a bus. His head hit the
concrete pavement. According to a witness Remund was unconscious for a while but regained his
consciousness as if nothing happened. However, after a while he complained of severe headache and
asked to be brought to the nearest Emergency Room. You are the nurse in the Emergency Room. If
increased intracranial pressure is suspected, what would be the sign?
a. Involuntary posturing
c. Papillary asymmetry
b. Irregular breathing pattern
d. Alteration in level of consciousness
108.
You are caring for Remund who sustained multiple injuries following an automobile accident.
Your initial assessment revealed that he is oriented to person and place but is rather confused as to
time. He complains of severe headache and drowsiness. His pupils are both equal and reactive to light.
Your critical nursing intervention would be:
a. Prevent unnecessary movement
b. Prepare to administer Mannitol
c. Keep Remund alert and responsive
d. Monitor for signs of increased intracranial pressure
42

109.
Berne is admitted into the emergency room following an assault where she was beaten in the
face and head. Based on Bernes history, which of the following interventions should be formed first?
a. Insert an oral or nasopharyngeal
c. Insert an intravenous catheter
airway
d. Obtain arterial blood gases
b. Give 100% oxygen by mask
110.
Marlou losses consciousness, you should prepare for which of the following FIRST?
a. Endotracheal intubation or surgical airway placement
b. CT scan on the head
c. Place a nasogastric tube
d. Place a second IV line
111.
112.
Situation 15- Lucilla is an Emergency Department nurse working during the morning shift.
A newly shock. Which of the following instructions of Lucilla to the newly hired nurse is LEAST
intended to obtain data regarding hypovolemic shock?
113.
A 41 year old victim of gunshot wound is being assessed closely for signs of hypovolemic shock.
Which of the following instructions of Lucilla to newly hired nurse is LEAST intended to obtain data
regarding hypovolemic shock?
a. Talk to the patient
b. Note skin color of the patient
c. Report to me changes in vital signs
d. Maintain pressure on the wound
114.
The newly hired nurse observed Lucilla perform assessment on a 50 year old female who
sustained partial and full thickness burns on both lower extremities due to fire. Which of the following
questions asked by Lucila will the newly hired nurse consider as an attempt to determine full thickness
burns?
a. Can you move both extremities?
b. How long were your extremities exposed to the flames?
c. Do you cover your extremities with any material like a blanket?
d. Do you experience pain?
115.
116.
117.
118.
Lucilla administered as prescribed, antivenom and tetanus toxoid to a client admitted with history
of snakebite. If you were the newly hired nurse. Which of the following statements will you consider
INCORRECT?
a. Tetanus toxoid enhances effect of antivenom
b. Amount of antivenom is dependent on the severity of reaction than weight of the client
c. Complications induced may be prevented by tetanus toxoid
d. Antivenom is an antidote for snakebite
119.
Lucilla instructed the newly hired nurse to inform the client with congestive heart failure to avoid
Valsalva type maneuvers. The newly hired nurse understands that these include the following EXCEPT:
a. Walking to and from the bathroom
c. Moving from supine to lateral position
b. Coughing and straining
d. Getting out of bed to a wheelchair
120.
When appraising the performance of the newly hired nurse during the shift, which of the following
behaviours will Lucille consider as reflective of a responsibility to improve evaluation ability?
a. Seeks clarifications regarding deviations from standard procedures
b. Organizes reference materials on medication prescriptions
c. Questions appropriately data obtained from the client
d. Asks for supervision of performance of a new procedure
121.
122.
Situation 16- Herminio, 70 years old, was admitted for operative repair of abdominal aortic
aneurysm.
123.
The nurse who admitted that patient recognizes that the development of abdominal aortic
aneurysm in the patient could be associated primarily with which of the following data obtained through
nursing history.
a. He stopped smoking when he was 65 years old
b. Patient is under treatment for hyperlipidemia
c. Patient is 70 years old
d. Prolonged history of hypertension
124.
When the nurse examines the patients abdomen, which of the following will she expect upon
palpation?
a. Pulsating mass on the mid and upper abdomen
b. Bruit over a mass in the abdomen
c. Rebound tenderness on the mid abdomen
d. Rigid board like abdomen
43

125.
Preoperatively, the priority nursing diagnosis in the nursing care plan is Risk for ineffective tissue
perfusion related to aneurysm rupture resulting to hemorrhage.Which of the following interventions is
most appropriate to prevent rupture?
a. Apply cold compress on the abdomen
c. Maintain complete bed rest with legs
b. Place patient on a Fowlers position
flat on bed
d. Relieve pain immediately
126.
Postoperatively, the nursing orders include monitoring patient closely for signs of graft leakage.
Which of the following signs of graft leakage will the nurse consider to be related to renal perfusion?
a. Decreasing hourly urinary output
c. Increasing pelvic and groin pain
b. Increased abdominal pain
d. Ecchymosis of scrotum
127.
Second day postoperatively, the patient complained of absence of sensation in the lower
extremities. Recognizing manifestations of complications, the nurse attributes the complaint to which of
the following?
a. Effect of anesthasia
c. Electrolyte imbalance
b. Spinal cord ischemia
d. Arterial occlusion
e.
f. Situation 17- Victoria, the staff nurse is preparing a teaching plan for Mrs. Santos a 75
year old who is recovering from an episode of Acute Bronchitis which exacerbated her
diabetes. Mrs. Santos is hard of hearing and arthritic but alert and oriented.
g.
128.
In developing the teaching plan for Mrs. Santos, which of the following steps is done after
Victoria has identified the learning needs of her client?
a. Determine content
c. Set learning outcomes
b. Set priorities for teaching
d. Organize the learning experiences
129.
Which of the following behavioural objectives is MOST appropriate before Mrs. Santos is
expected to self administer medications prescribed by the physician?
a. Write the names of the drugs
c. Select the prescribed medications
b. Identify all the medications
d. Organize the medications
130.
131.
132.
133.
134.
135.
During assessment, which of the following is best for Victoria to do to be able to determine the
learning style preferred by Mrs. Santos? Ask Mrs. Santos:
a. The things she usually do
d. Who will be interested to learn with
b. How she learned best in the past
her
c. For changes she is willing to do
136.
This of the following will be MOST effective when Victoria uses the one on one discussion
method of teaching?
a. Frequently rephrase statements to facilitate understanding
b. Use printed materials with all capital letters for easy reading
c. Allow Mrs. Santos to recommend a schedule of drug administration
d. Limit to verbal instructions
137.
During the discussion, Victoria asked Mrs. Santos to repeat what she just taught Mrs. Santos did
not respond. Which of the following is BEST for Victoria to say?
a. Are there things which I did not say clearly
b. Mrs. Santos, did you hear what I ask you?
c. Did you understand what I have just taught you?
d. I asked you to repeat what I have just said, Mrs. Santos?
138.
139.
Situation 18- You are assigned in the Cancer Institute and caring for 5 patients with
varying types of cancer.
140.
141.
Hector, non-Hodgkins lymphoma, develops a platelet count of 10,000/ul during chemotherapy.
Base on these findings, an appropriate nursing intervention is to:
a. Encourage fluids to 3000ml/day
c. Check the temperature q 4hr
b. Check all stools for accult blood
d. Provide oral hygiene q 2 hr
142.
Fannie, a 26 year old teacher with stage II Hodgkins lymphoma asks you How long do I have to
live? Your BEST response is:
a. Most patients with your stage Hodgkins disease are treated successfully
b. It will depend on how your disease to radiation , but most patients do well
c. You know , no one can predict how long someone will live , so try to focus on the present
d. With ongoing maintenance chemotherapy, the 10 year old survival rate is very good
143.
Debra, who has ovarian cancer, tells you, I dont think my husband cares about me anymore. He
rarely visits me. During the visit of Debras husband you greeted him and he told you.I could not stand
44

to see my wife so ill and I dont know what to say to her What will be your APPROPRIATE nursing
diagnosis in this situation?
a. Interrupted family processes related to effect of illness on family members
b. Compromised family coping related to disruption in lifestyle and role changes
c. Risk for caregiver role strain related to burden of care giving responsibilities
d. Impaired home maintenance related to perceived role changes
144.
Jenny 40 year old single mother of two school age children is hospitalized with melastatic cancer
of the ovary. You find her crying, and she tells you that she does not know what will happen to her
children when she dies. Your most appropriate response is:
a. For now you need to concentrate on getting well. Do not worry about your children
b. Why dont we talk about the options you have for the care of you children?
c. Many patients with cancer live for a long time, so there is time to plan for your children.
d. Perhaps your ex husband will take the children when you cant care for them
145.
When assessing Lermas needs for psychologic support after she has been diagnosed with
stage I cancer of the colon, which questions will you ask to give you the MOST information?
a. Are you familiar with the stages of emotional adjustment to a diagnosis like cancer of the colon
b. How long ago were you diagnosed with this cancer?
c. How do you feel about having a possibly terminal illness?
d. Can you tell me what has been helpful to you in the past when coping with stressful events?
146.
147.
Situation 19-Luis, 58 years old was admitted for surgical repair of an aneurysm in the
ascending aorta detected through an annual physical examination required by the company
where Luis is currently employed.
148.
149.
Which of the following questions should the nurse include when conducting a health history to
identify a genetic risk of aortic aneurysm?
a. Have you experienced neck and back pain?
b. How long have you been hypertensive?
c. Did any of your children have hypertension or ant heart problem?
d. Has aneurysm ever been diagnosed in your family?
150.
151.
152.
153.
154.
Physician confirmed his diagnosis that Luis aortic aneurysm is the fusiform type of true
aneurysm. The nurse recognizes that this type of aneurysm:
a. Is spindle shaped and involves three layers of the vessel wall
b. Is characterized by a tear in the tunica intima with blood invading layers of the vessel wall
c. Involves the entire diameter of the ascending aorta
d. Is shaped like a sac involving only one layer of the vessel wall
155.
As the client is being prepared for surgery, which of the following assessment data should the
nurse report IMMEDIATELY to the physician?
a. Positive abdominal reflex
c. Abdominal bruit
b. Increased white blood cell count
d. Elevated blood pressure
156.
Postoperatively , nursing orders include close observation for signs of arterial thrombosis, which
of the following assessment data should you NOT consider as a manifestation of this complication?
a. Absence of peripheral pulses
c. Increased abdominal girth
b. Increasing abdominal and scrotal
d. Elevated body temperature
pain

45

157.
Protamine sulphate was prescribed by the physician as a PRN medication. The nurse
understands that this prescription is intended to
a. Reverse effects warfarin therapy
b. Serve as an anticoagulant for arterial occlusion
c. Counteract adverse bleeding due to heparin therapy
158.
159.
Situation 20- A nurses perform their responsibilities in their job, they encounter situations
which expose them to possible lawsuits.
160.
161.
The nurse on night duty assumed that her patient is sleep. At the bedside she discussed in detail
the condition of the patient with another nurse. Which of the following may be a possible legal complaint
against the nurse?
a. Poor bedside behavior
b. Breach of confidentiality
c. Breach of privacy
d. Negligence
162.
An 85 year old woman diagnosed with Metastatic Breast Cancer, she has a colostomy and a
fractured hip. The night nurse documented that she became delusional and restraint was applied. No
relative could say with the patient during the day. The incoming nurse recognizes possible liability in the
application of the restraint. Which of the following will she do FIRST?
a. Discuss concerns with relatives
b. Obtain a medical order for a sedative
c. Consult attending physician
d. Select a more appropriate and safe restraint
163.
After cardiac catheterization, a female patient developed chest pain and died. The family sued
the hospital contending that. It had a duty to inform the patient of the risk of the procedure. The court
decided in favour of the hospital. Which of the following is MOST likely the basis of the courts decision?
a. The physician is the best person to inform the patient about the procedure
b. The representative of the hospital was not present during the procedure
c. The hospital practices quality control
d. The informed consent was signed by the patient
164.
Elizabeth is a registered nurse who works in call center. Because she is a nurse, her co-workers
asked her to administer their allergy hormone medications. Which of the following actions best to avoid
liability?
a. Administer the medication after duty hours in the call center
b. Encourage co-workers to learn self-administration of drug
c. Administer the medication and dose correctly
d. Tell co-workers to secure a written medical prescription with physicians license number
165. An elderly patient refused to take her cardiac medication. Thinking that she was disoriented , the
nurse crushed the tablets and mixed them with the soup and fed the patient. The nurse may be held
liable for:
a. Negligence
b. Battery
c. Assault
d. Violation of privacy
166.
167.
168.
169.

170. A1 PASSERS TRAINING, RESEARCH, REVIEW AND DEVELOPMENT COMPANY


171.
PRE-BOARD EXAMINATION
172.
173.
NURSING PRACTICE V-Care of Clients with Physiologic and Psychosocial Alteration
174.
175.
176.
GENERAL INSTRUCTIONS:
177.
1. This test booklet contains 100 test questions.
178.
2. Read INSTRUCTIONS TO EXAMINEES printed on your answer sheet.
179.
3. Shade only one (1) box for each question on your answer sheets. Two or more boxes shaded will
invalidate your answer.
180.
4. AVOID ERASURES.
181.
182.
INSTRUCTIONS:
183. 1. Detach one (1) answer sheet from the bottom of your Examinee ID/Answer Sheet Set.
184. 2. Write the subject title Nursing Practice I on the box provided.

46

185. 3. Shade Set Box A on your answer sheet if your test booklet is Set A; Set Box B if your test booklet is
Set B.
186.
_________________________________________________________________________________
_________
187.
188.
SITUATIONAL
189.
190.
Situation 1- It is the first day of Orientation of new nurse at the Psychiatry Ward. The
supervising nurse emphasizes that it is the responsibility of all members of the interdisciplinary team
to safeguard patients records from loss of destruction or from people not authorized to read it.
1. It is unethical to tell ones friends and family members data about patients because doing so is a
violation of patients rights to:
a. Informed consent
c. Civil liberty
b. Confidentiality
d. Secrecy
2. The nurse must see to it that the written consent of mentally ill patients must be taken from
a. Law enforcement agencies
c. Doctor
b. Social worker
d. Parents or legal guardian
3. In an extreme situation and when no other resident or intern is available, should a nurse receive ,
telephone orders, the order has to be correctly written and signed by the physician within:
a. 24 hours
c. 12 hours
b. 48 hours
d. 36 hours
4. To facilitate identification of persons relationship, the family nurse utilizes this diagrammatic
representation of members of a family and their relationships
a. Kardex
c. Genogram
b. Flow chart
d. Algorithm
5. The MOST significant event the nurse records regarding psychosocial well being are
a. The patient sleeping , eating and elimination patterns
b. The behavior patterns and interpersonal interactions of the client
c. Somatic treatments, medications and their effects
d. Signs and symptoms and physical appearance
6.
7. Situation 2- Ms. Corpuz, the psychiatric nurse at the family section of the Out-Patient Department
follows up families of discharged schizophrenic patients. These follow up visits provide
opportunities for psychosocial treatment of patients and their families.
8. To ensure maximum participation of family members in follow up sessions with Ms. Corpuz. Which of these
elements of effective family intervention is basic?
a. Family social support
c. Training in problem in problem solving
b. Improvement in family communication
d. Mutually agreed upon goals
9. The primary goal of psychosocial treatment for families of discharged psychiatric patients as
a. Provide gainful employment
b. Ameliorate poverty and social ills in the community
c. Provide education on medication management
d. Enhance coping efforts and reduce stressful life events
10. Which of the following is a cultural factor that a barrier to avail of family intervention?
a. Resources and time
c. Shame and stigmatization
b. Unresponsive government policies
d. Traditional practices
11. All of these are within the scope of the generalist nurses role, EXCEPT:
a. Social skills training
c. Family therapy
b. Individual and family assessments
d. Health education on psychotropic drugs
12.
13.
14.
15.
16. Cognitive behavior principles are utilized in psychosocial treatment. In such interventions patients who
are rehabilitating from schizophrenia are.
a. Taught to unlearn maladaptive behaviours through desensitization
b. Assisted to work through their unconscious internal conflicts
c. Taught to reframe psychotic symptoms as coping attempts rather than signs that they are crazy
or weak
d. Made to recognize the importance of psychotropic drugs to combat delusions and hallucinations
17.
18. Situation 3- Joe, 30 years old was admitted to the Psychiatry Ward because of changes in
behavior such as neglect of self care. Withdrawal from relations with people talking to himself
and beliefs that he is being persecuted. He has been diagnosed with Schizophrenia disorder.
19.

47

20. Nurse Gina was assigned to take care of Joe. She approached and greeted Joe and sat with him to
start interaction. He moved back to distantiate himself and evaded eye contact. Nurse Gina
recognized that in order to establish a nurse-patient relationship, it is important Joe to have:
a. Self confidence
c. Rapport
b. Self worth
d. Trust
21. Nurse Gina observed that Joe kept mumbling unintelligible words which made no sense to her
accompanied with inappropriate facial grimaces as if he is talking to someone. He claims he is in a
prison camp. Nurse Gina interprets these behaviours as
a. Depersonalization
c. Lack of insight
b. Anhedonia
d. Ambivalence
22. The multidisciplinary team shared observations of Joe and discussed plans for treatment Nurse Gina
anticipates that from evidenced based practice, the MOST likely treatment plan to be initiated is
a. Behavior modification
c. Remotivation therapy
b. Relationship therapy
d. Pharmacotherapy
23. A socialization program is scheduled for the day. A therapeutic intervention of Nurse Gina is:
a. Minimize environment stimuli and have Joe engage in ales stimulating activity
b. Present the plan for the day and have Joe decide
c. Have a friendly invite Joe to the socialization program to overcome his withdrawal from social
relations
d. Refer the matter to the occupational therapist
24. A therapeutic attitude in dealing with Joe is by being
a. Lenient and have no demands
c. Simply objective and businesslike
b. Actively friendly with him
d.
e. Situation 4- The nurse is caring for an elderly client who has a severe hearing impairment
25. Which of the following is the MOST important part of assessing a client with hearing loss?
a. Change in style of communication
d. Use of simple noninvasive way to test
b. Weariness or unexplained irritability
hearing
c. History of a client
26. The nurse is preparing a nursing history . in communicating with the client, which of the following is the
MOST appropriate technique?
a. Speak loud enough to be heard
c. Sit bedside the client with the affected
b. Speak directly and clearly facing the
ear
client
d. Use cardboard when asking question
27. The nurse is performing a physical exam on the client. Which of the following is the MOST appropriate
method to be used by the nurse?
a. Bone conduction
c. Audiogram
b. Whisper test
d. Otoscope
28. An elderly client asked the nurse why older adults are more prone to conductive hearing loss and
tinnitus. The nurse addresses appropriately the query of the client when she states that it is important to
INITIALLY rule out:
a. Injury to the middle air
b. Hardened cerumen lodged in the external ear
c. Damage to the tympanic membrane
29. Which of the following is an appropriate nursing intervention for the client with hearing impairment?
a. Get an interpreter every time the nurse makes contact with the client
b. Prepare set of questions and responses of client
c. Minimize environment noise before speaking with the client
d. Place the client in a quiet well ventilated room
30.
31.
32.
33.
34.
35. Situation 5- Nurse Flerida, the Community Health nurse, attended a Barangay meeting where
problem of Troubled teenagers who are delinguents, possible drug and alcohol abusers and
get involved in occasional violence.. was discussed. She approached Gerry, the President of
Sanggunian Kabataan and they talked about developing a strategy which resulted in series of
meetings with teenagers in the Barangay.
36. An informal first meeting with some teenagers was held in basketball court in order to engage the
teenagers to talk about their concerns, it was essential for Nurse Flerida and SK Gerry to:
a. Establish a contract with them
b. Ensure confidentiality of their discussions
c. Avoid bringing any gadget that the teenagers can get suspicious of
d. Just listen more than talk
48

37. On the third meeting , 20 teenagers showed up. The teen started to talk about their problems and about
marijuana, acid , experimenters, frequent users, pushers, etc.. at this point. It would be therapeutic for
Nurse Flerida and Gerry to:
a. Make effort offer solutions or answers to their problems
b. Practice moralistic judgment
c. Maintain active listening
d. Translate t the teenagers what is going on in their world
38. The themes of the conversations with the teenagers centered around, No one ever listens, and We
tried to talk with our teacher, then the guidance counsellor, the other teachers , the coach and still
others but they just dont have time.. The nurse infers that these young people:
a. Are having reaction formation
c. Desperately want relationships with
b. Primarily want to avoid prison
adults
sentences
d. Worry about parental punishment
39. During the fourth meeting 40 adolescents attended, all were eager to talk, share experiences an offer
ideas. The adolescents asked about the personal lives of Nurse Flerida and Gerry and who they had
shared information about their situations. This must be taken as due to:
a. Develop objectivity
c. Deepen trust in the relationship
b. Maintain professionalism
d. Establish control of the situation
40. After the fourth meeting, Nurse Flerida and Gerry explored ideas about future directions. In bringing in a
community leader to listen and talk to these teenagers, the priority objective of such meeting would be to:
a. Avoid confinement in mental hospitals
d. Do health screening among the
b. Explore options and solve problems
teenagers
c. Circumvent heavy prison sentences
e.
f. Situation 6- A non- experimental , one group, pretest postiest research design was used
to evaluate the effectiveness of a newly developed training program for nurses working
with electroconvulsive therapy (ECT) for psychiatric patients.
41. Subjects were selected because they happen to be available for participation in the study the process
used is
a. Random sampling
c. Probability sampling
b. Convenience sampling
d. Purposive sampling
42. The study was approved by the health Services Ethics Committee. Prior to completing questionnaire at
the start of the training program, the consent would be provided by:
a. Nurse supervisors
c. Hospital medical director
b. Patients undergoing ECT
d. Nurse participants
43. An evaluation questionnaire included 14 True-False items which measured Nurses knowledge of key
components of providing assistance in an ECT procedure. In this study participants knowledge were
measured.
a. Solely after the training
c. Before and after the training
b. Solely before the training
d. In the middle of the training
44. Six items of the evaluation questionnaire used a 10 point scale( ranging from not all confident to very
confident) to measure nurses perceived level of confidence with setting up an appropriate ECT
machine. The type of scale used is
a. Interval scale
b. Ordinal scale
c. Ratio scale
d. Nominal scale
45. Table changes in Participants Knowledge Before and After the ECT Program (N=92) In the table where
N-92, N=92 means
a. Percentage of nurses responding correctly in the post test
b. Number of nurses who participated in his study
c. Number of nurses currently working in the field of ECT
d. Percentage of nurses who completed both pretest and posttest questionnaire
46.
47.
48. Situation 7-Psychiatric nursing practice has evolved from an institutionalized setting to a more
humane approach. Ideas about mental illness, human nature and the environment continue to
challenge roles of nurse.
49. Psychiatric treatment encourages clients independence. The nurse recognized which of these setting to
be LEAST restrictive
a. Half-way homes
c. Family of orientation
b. State hospitals
d. Nursing homes
50. As the date for discharge approaches, a client becomes increasingly anxious and regresses, thus
delaying staff decision to discharge client. The client behaviour following this decision, improves and
this pattern gets repeated. The staff decision to delay discharge acts as a.
a. Negative reinforce
b. Conditioner
49

c. Punishment
d. Positive reinforcer
51. Chronic schizophrenic patients are assisted to learn self help behaviour by way of tokens for good
grooming. Which in turn are used to present at dinner to be served their meals . this is an intervention
used in
a. Social therapy
c. Behaviour modification
b. Relationship therapy
d. Community involvement
52. Client is psychotic and confused, priority in planning is to
a. Provide structural controls
c. Maintain safety of other patients
b. Maintain client safety
d. Avoid damage to environment
53. Which of these opportunities will the nurse create to teach clients a sense of responsibility and learn the
consequences of their actions?
a. Remotivation groups
c. Client government
b. Activity groups
d. Discussion groups
e.
f. Situation 8- The nurse in a half way house where children at risk are taken care of the
following situations are encountered
54. At night the children were being prepared to go to sleep, the nurse heard from a frightened child
AYAW KO MATULOG MAG-ISA, MAY MULTO! (I dont want to sleep alone Theres a ghost!) the
nurse assess this behaviour as need for
a. Biological integrity
c. Safety and security
b. Love and belongings
d. Trust
55. Most street children come from broken families. The most effective role for the nurse to facilitate
development of trust is
a. Mother surrogate
c. Child therapist
b. Teacher
d. Activity therapist
56. When a child is called her name, clothed appropriately and given age appropriate tasks , she:
a. Develops a sense of competence
c. Develops sense of identity
b. Learns to trust in self and in others
d. Acquires a sense of personal power
57. Preventive interventions for children at risk are BEST achieved through
a. NGOs and other workers paying attention in conflict ridden or problematic community
b. Support and caring to children during family crisis situations
c. Play and activity therapist
d. Family therapy for the dysfunctional families
58. Mina was a frequent witness to domestic violence. Her father would always come home drunk and beat
up Minas mother. As an effect of this experience she had nightmares speechless for weeks; inability to
sleep, tension and palpations lasting for more than a month. Mina is experiencing:
a. Anxiety disorder
c. Post traumatic stress disorder
b. Adjustment disorder
d. Phobic disorder
e.
f. Situation 9- Kate is 9 year old. Horribly sexually abused by her father, she had developed
a way of behaving: she trips naked. Urinates on the floor and vomits. For some reason,
she thinks it is alright for her to strip: urinate on the floor and hit and kick on the staff,
and then asks sweetly for a hug. She is the one of the kids on the child and Adolescent
Unit of the hospital.
59. Form the family history of Kate the nurse infers that a fundamental issue is Kates
a. Lack of maternal care an warmth
b. Feelings of hostility of own family
c. Inconsistent emotional and physical relationships
d. Inability to trust relationships
60. With other behaviourally disturbed children in the unit. Kate smiles with glee watching adult staff
struggle to take time to fix the place and put things in order every time these kids create chaos. This
pattern of behaviour is.
a. Hostile
c. Manipulative
b. Assaultive
d. Demanding
61.
62.
63.
64. Kate has developed hostile attitudes as shown in her hitting and kicking behaviour. The
psychodynamics of Kate behaviour most likely would point to
a. Inability to cope with conflict and frustration
b. Lack of role models in her growing up
c. Cry for plea for protection of children rights
d. Inability to discern what is acceptable from unacceptable behaviour
65. In creating a therapeutic environment for Kate and kids of the same situation , it is FOREMOST to:
a. Develop a positive self image
c. Provide values education
b. Provide therapeutic limit setting
d. Provide values education
50

e. Observe physical hygiene and adequate nutrition


66. In the unit, it is uncommon to hear loud bangs and thuds with yelling, screaming and cursing of these
children with the company of Kate. A therapeutic activity that the staff can provide is
a. Sports that foster cooperation and teamwork
b. Painting and art work
c. Active ball games that are not competitive
d. Organizing scrapbook clubs
e.
f. Situation 10- Roger, 24 year old, sustained fracture of the right tibia and fibula following a
motorcycle accident. The physician ordered application of long leg cast.
67. The physician applied to cast to immobilize the fracture leg. The plaster of pairs cast is damp and the client
verbalized that his leg feels very hot. What should be the appropriate action of the nurse?
a. Reassure the client that it is a common effect after the application of the cast
b. Elevate the limb to facilitate drying of the cast
c. Tell the client that he will experience feeling hot for several hours as the moisture evaporates and the
cast hardens
d. Notify the physician as the cast is constricting the legs, causing so much pressure
68. To facilitate the drying of the long leg cast, the nurse should do which of the following measures?
a. Elevate the affected leg on a bed board
c. Place the bed in cool place
b. Instruct the client to assume one position
d. Expose the cast fully to the air
69. The nurse is assessing the client with long leg cast. She noticed that the clients toes are pale, cool and
capability refill is delayed for five seconds. How should the nurse interpret this finding?
a. The nerves in the toes are threatened
b. Stasis of venous blood in the toes
c. The finding is normal for this recovery time period
d. Decrease of arterial blood supply in the toes
70. Several hours after the application of the long leg cast, the nurse notices that the client toes are edematous,
the nurse refers her observation to the attending physician, the physician decided to bivalve the cast. The
client asks the nurse what the procedure is. The nurse explains to the client that this procedure is done by:
a. Splitting the cast near the toes to relieve constriction
b. Placing the client leg in a dependent position
c. Splitting and spreading the cast on each side
d. Cutting the cast and reapply after 24 hours
71. The nurse is preparing the client to ambulate. To prepare the client, the nurse should encourage the client
to:
a. Sit up in bed for 30 minutes twice a day
b. Keep the affected limb in straight alignment
c. Keep the affected limb in straight alignment
d. Practice sitting using a trapeze to strengthen muscles
e. Perform exercise in bed to strengthen the upper extremities
72.
73. Situation 11- Carlyn , is a 28 year old, female patient, admitted to the psychiatry unit with a diagnosis
of chronic undifferentiated schizophrenia. She describes herself as Binibining Pilipinas
Universeand her mission is to represent the country and be an ambassador of good will for the
Filipino people. She was observed to be talking a lot and laughing by herself. Her thoughts were
incoherent and disorganized. She was also aloof in relating with others
74. Carlyn claim of being Binibining Pilipinas Universe is an:
a. Delusion
c. Obsession
b. Imagination
d. Illusion
75. The nurse interprets Carlyn thought of being an ambassador of good will for the Filipino people as a
a. Attempt to overcome low self esteem
b. Mechanism to connect with reality
c. Personal vision that is realistic and laudable
d. Regressive behaviour
e.
f.
g.
h.
i.
76. Which of these interventions would NOT be therapeutic in decreasing Carlyns anxiety?
a. Listen to her thoughts and feelings
b. Joke about her thoughts to help her fetal at ease
c. Do not challenge her perception
d. Simply accept
77. Which of the following is a PRIORITY intervention for the nurse?
a. Activity therapy
c. Behaviour modification
b. Milieu therapy
d. Relationship therapy
78. In order to assist Carlyn to cope and be prepared for discharge, it is BEST for the nurse to focus on:
51

a.
b.
c.
d.

Insight into personal problems


Attitudes of society towards the mentally ill
Skills for maintaining daily living
Awareness of interpersonal interactions

79.
80. Situation 12- Mr. B was a post-operative patient who died from injuries sustained in a fall from
his third floor window. Apparently, he had tried to climb down on an improvised rope. At trial the
nurse testified that during her last evening visit with him, he had experienced an episode of
tachycardia and hypertension, refusing all nursing care and prescribed medication known to
have adverse effects including confusion and anxiety. The nurse did not report to the physician
and testified also that when she passed by the patient at midnight, he appeared to be sleeping
and did not reassess his vital signs.
81. The nurse negligence includes
a. Failure to document
d. Failure to assess and monitor and
b. Failure to follow standards of care
failure to communicate
c. Failure to act as patient advocate
82. Failing to maintain a safe environment in this instance is a joint accountability of the nurse and the
a. Nursing staff
c. Hospital
b. Doctor
d. Security personnel
83. When the nurse testified that the patient appeared to be sleeping , which of these competencies did
she fail to do?
a. Inspect
c. Analyze
b. Plan
d. Observe
84. Full documentation of care on the patients chart in addition to being factual, accurate, and complete
includes being
a. Thorough
c. Tidy
b. Truthful
d. Timely
85. When this incidence of fall is reviewed, what is the primary basis for considering this to be a nursing
negligence?
a. Nurse practice act
c. Nursing procedure manual
b. Nursing supervisor evaluation
d. Hospital rules and policies
e.
f. Situation 13- Nurse Linda is a young promising nurse who chose to start a professional
career as a mental health psychiatric nurse. Part of her orientation and training as a
beginning professional nurse is enhancing facilitate personal characteristics.
86. Nurse Linda recognizes that therapeutic effectiveness is BEST acquired through
a. Human relations laboratories
b. Education in the behavioural sciences
c. Workshops like sensitivity sessions
d. Knowledge and practice of specific interpersonal skills in everyday life
87. Anne is a new client of Nurse Linda. She offers Linda a gift. Nurse Linda responds therapeutically by:
a. Clarifying clients intent, I wonder why you are offering me a gift.
b. Reacting emphatically No it is unethical to receive gift from a clients
c. Citing a hospital policy, It is against hospital policies.
d. Accepting with an acknowledgement, I feel uncomfortable doing this but I dont want to
disappoint you.
88. Who is the following clients must Nurse Linda refrain from use of touch?
a. Marissa who is aggressive and hostile
b. Rodolf who is depressed with low level of energy
c. Agom who is looking for a mother
d. Mico who has very low self esteem
89. Resistance is often mistaken seen as the clients struggle against
a. Change
c. Responsibility for actions
b. Self-awareness
d. The nurse
90.
91.
92.
93.
94. Establish a therapeutic contract is the goal of which phase of the nurse patient relationship?
a. Orientation phase
c. Pre orientation phase
b. Working phase
d. Middle phase
e.
f. Situation 14- Karen, a19 year old nursing student, faints in the Ladies Room of the
University Bookstore; a staff person discovers her and calls an ambulance. In route to
the Emergency Department, she drifts in and out of consciousness. Her blood pressure.
52

70/42 mmHg is barely audible; pulse 62 beats per minute, erratic and difficult to palpate.
When she arrives at the ED, an IV of 1,000 D5W is started Karen is conscious but weak.
95. Karen admitted to the nurse that she took about a handful of laxatives over the course of 6 hours...to
which the nurse responded Thatsnot too bright. The chart says youre a nursing student. You should
know better than that, hmm?the approach is
a. Accepting because she is just stating an information from her chart
b. Encouraging client to use her judgment appropriately
c. Simply a matter of fact attitude
d. Not therapeutic because she is admonishing
96. Karen is diagnosed to have Bulimia Nervosa. This characterized by binging and typical behaviour is
a. Excessive anxiety of hospitalization and encountering health practitioners that probe into her
private life
b. Eating larger amounts of food than most people do under similar and circumstances
c. Uncontrolled urge to seek secondary gain from other people
d. Seating unusual and non nutritive kinds of foods and substances
97. A nurse counsellor talked to Karen after her emergency needs have been attended to which of the
following questions should the nurse avoid:
a. How often are you moving your bowels?
b. Have you tried to make yourself throw up to relieve yourself of stomach distress?
c. Why do you take laxatives to feel thinner or to loss weight?
d. Do you sometimes take something to move your bowels along?
98. Clients like Karen their illness because of:
a. Pretending to seek help
c. Uncontrolled urge
b. Fear of being judged
d. Non threatening nature of the illness
99. The long term goal of the nurse counsellor is
a. Educate on personal and mental hygiene
b. Increase self esteem essential to health
c. Develop trusting relationships
d. Educate on proper food servings and dietary requirements
100.
101.
Situation 15- Television and newspaper report that drug abuse is a serious social,
economic and political problem in our society. Nurses are continually challenged to play a role
in abetting in the community
102.
Street children are introduced early to harmful substances such as inhaling rugby. This
observation underscores the need to address the problem at its core. Nurses contribute best at the
preventive level by promoting:
a. Social welfare for under privileged families
b. Responsible parenthood
c. Family planning
103.
Teen agers who join fraternities and gangs are high risk groups for substance abuse which
psychosocial need is best met through per counselling?
a. Recognition
c. Belonging
b. Self esteem
d. Security
104.
Benjie is using marijuana, which of the following behaviours would be observed?
a. Watery eyes, rhinitis, sneezing , yawning
b. Appears drunk with staggering gait
c. Uses dark glasses to conceal bloodshot eyes
d. Appears alert and confident with dilated pupils
105.
A college student client states that his addiction to morphine is due to a chemical imbalance. The
nurse understands that researches that point to biochemical theory of addiction support that drugs of
abuse have in common , stimulation of which of the following neurotransmitter?
a. Norepinephrine
c. Acetylcholine
b. Serotonin
d. Dopamine
106.
107.
108.
109.
110.
111.
At the Emergency Room, a young guy was brought in and succumbed to sudden heart attack.
The parents reported that their son had always been healthy. a report was made that substance use
was positive. MOST commonly abused substance by this age is
a. Marijuana
c. Cocaine
b. Steroids
d. Alcohol
e.
f. Situation 16- The following are behavioural interactions which have occurred within the
ward milieu in which patient and staff have encounters
53

112.
While the patients were gathering in the gym for the morning exercise, Jamelie approached the
nurse and reported about a bad behaviour of another patient. The nurse observed Jamelies repeated
pattern of brining messages about misbehaving patients. The nurse interprets this correctly as a/an
a. Compulsive behaviour
b. Policing behaviour which helps to ensure discipline in the ward
c. Attempt to enhance her power with the nurse
d. Matter of fact behaviour that does not necessarily have any intended meaning
113.
During the ward meeting. Dan interrupts to ask the nurse. What time is it? Even though that
wall clock is visible the everybody. it is best for the nurse to
a. Have him refer t the clock
c. Respond, Am I your time keeper
b. Ignore this behaviour
d. Simply tell the time
114.
During the Nurse-patient interaction, the patient remarked without trigger, it bothers me as he
showed disturbed behaviour. The nurse responds therapeutically when she
a. Observes and remains client
b. Asks, when you say it what are you talking about?
c. Verbalize back, you are bothered
d. Reflects back, You are bothered
115.
Edward has been noted to have the repeated pattern of relating his thoughts as you know or
everybody knows.It is the best for the nurse to
a. Interrupt as this may lead to obsessive thinking
b. Take this as simply a matter of expression
c. Observe for ideas of reference
d. Respond by asking details
116.
Virgilio, a newly admitted patient claimed, they are talking about me as began to be upset and
irritable . a therapeutic response of the nurse would be to
a. Ask who are they?
d. Allow him to continue his thought
b. Say, tell me what is upsetting you?
process
c. Inquire on what is being talked about.
e.
f. Situation 17- The nurse wants to develop a client education program to increase the
outpatient client s knowledge and skill regarding medication management
117.
Effectiveness of this program is BEST demonstrated with the clients
a. Rigidly following doctors orders regarding drug intake
b. Self reliant, self care behaviors and responsible use of prescribed medications in the community
setting
c. Religiously recording the intake of their medications
d. Increased frequency of communication with the health care team to consult for any problem that
may arise
118.
The INITIAL goal this program would be
a. Create more productive alliance between the client and members of the interdisciplinary mental
health care team
b. Increase treatment compliance
c. Impart the information necessary for independent medication management
d. Teach responsible problem solving and decision making skills regarding medication
119.
Which of the following steps reflects the client awareness of the first step in the correct
procedure to be used when taking medications?
a. Open the bottle and pour the correct amount into the bottle cap or other container
b. Read the label or the doctors orders carefully
c. Take the medication with a full glass juice or water
d. Close the bottle tightly and double check the doctors order
120.
You want to have the client demonstrate assertiveness skill in communicating with the health
care team. An appropriate method for immediate feedback is:
a. Nurse patient relationship
c. Psychotherapy
b. Role play
d. Counselling
121.
122.
123.
124.
125.
Which of the following offer social support regarding medication compliance?
a. Use a calendar to tract medication usage
b. Keep a journal or record medications
c. Have the doctor recommend the use of affordable generic medications
d. Have family and friends attend a medication education group with the client
126.
127.
Situation 18- Maria, 67 year old has a history of mental illness in her middle age and treatment
for early menopause.Medications were given for depression which brought her into emission; she
54

functioned fairly well but was prone to paranoid ideation, social isolation and became severely
anxious when stressed. She was brought to the hospital due to profound depression with mood
changes manifested by little sleep, high energy, hysterical laughter, and bizarre behaviour.
128.
Lithium was initiated, the nurse was effective in her health teaching when the client remarked:
a. I need to lessen my calorie and salt intake but have enough fluids
b. I need to have regular diet with adequate sodium and fluid intake
c. I should restrict my salt and fluids
d. I should sweat a lot with exercise to reduce my body fluids
129.
Maria claimed that she had been hearing frightening motorcycle noises, angry voices and sounds of
rushing floor water. The priority need at this point is
a. Trust
c. Self esteem
b. Love and belongingness
d. Safety and security
130.
Maria was started on antipsychotic medication. The following are effects of hatoperidol EXCEPT:
a. Akathisia
b. Pseudoparkinsonism
c. Control the affective symptoms
d. Voices decreased in both intensity and content
131.
The nurse watches for early lithium toxicity. Which of the following must be part of health teaching?
a. Seizures and anuria
c. High fever and tachycardia
b. Fine tremors, nausea, vomiting and
d. Chuffing gait and restlessness
diarrhea
132.
When agranulocytosis is noted, antipsychotic medication should be withheld and the patient is
watched for
a. Changes in sensorium
c. Hypertension
b. Involuntary muscle movement
d. Signs of infection

55

e.
f. Situation 19- management of spinal cord injuries vary from the management phase through
rehabilitation. The nurse faces challenges how to maximize health status to preserve quality
of life.
133.
Nurse Carol is assigned in the Neurological Ward. She is talking care of Bobby with spinal cord injury.
The goals of management for Bobby should include which of the following
a. Skeletal fracture reduction and traction , and pharmacological therapy
b. Continuous monitoring of vital signs and immobility
c. Prevent further injury and observe for symptoms of progressive neurological deficit
d. Pharmacological and physical therapy
134.
Nurse Carol is testing the motor ability of Bobby. She performs this procedure by
a. Asking the client to move his upper and lower extremities
b. Using a wasp of cotton from the shoulder down the lower extremities
c. Gently pinching the skin with a tongue blade staring at the shoulder level down to both lower
extremities
d. Asking the client to spread the fingers, squeeze the nurse hand and move the toes or turn the feet
135.
In taking care of client with spinal cord injury, the nurse MUST observe which of the following nursing
interventions?
a. Promoting adequate breathing , improving mobility, adaptation to sensory and perceptual alteration,
maintaining skin integrity and elimination, and providing comfort measures
b. Maintaining immobility, adequate nutrition and elimination, comfort measures pharmacological and
respiratory therapy to adapting sensory and perceptual alterations
c. Improving elimination , adapting to sensory and perceptual alteration pharmacological and respiratory
therapy and maintaining skin integrity
d. Adapting to sensory and perceptual function, maintaining immobility , providing comfort measures
and improving bowel functions
136.
One of the client assigned to nurse Carol is Norman, who is in Halo Traction. While administering the
8:00 A. M. Medication, she notices that one of the pins was detached, which of the following should be her
INITIAL action?
a. Ask assistance from the other staff nurses to immobilize the client leave the client in his position and
call for help with other staff
b. Leave the client in his position and call for help with other staff
c. Call the neurosurgeon
d. Stabilize the head in a neutral position and ask another nurse to notify the neurosurgeon
e.
f.
g.
h.
i.
137.
Nursing care of Norman with Halo Traction requires meticulous care under the halo vest which of
the following nursing measures MUST be done in providing care to Norman?
a. Inspect for excessive perspiration , redness and skin blistering, wash the torso and change liner
periodically
b. Inspecting the skin for redness and blistering , and open the vest periodically to promote comfort
c. Wash the torso and leave the liner in the body of the client and apply powder especially on the
bony prominences
d. Applying powder inside the vest to prevent skin breakdown after washing the torso
e.
f. Situation 20- Mr. Lazo, a 78 year old male was admitted to the hospital to rule out any
injury that may have resulted from a fall while going up the stairs of his home. He has no
recollection of what caused him to fall. While in the hospital, Mr. Lazo exhibited mental
status changes including disorientation and confusion.
g.
138.
Which of these questions would the nurse ask to gather data about Mr. Lazos orientation status?
a. Ask client the brothers and sisters names, children and grandchildrens names
b. Ask the client to repeat 6 to 7 digits forward
c. Ask clients name, address, current location, date, time and year
d. Name of 3 objects (e.g. , table , spoon, shirt) and have client repeat the three objects ask the
client to repeat them again after 5 minutes.
139.
In order to prevent complications for which Mr. Lazo may be at risk. Which of these will the nurse
prioritize in planning and outcome identification? Mr. Lazo will:
a. Will maintain existing ability to perform activities of daily living
b. Remain safe and free from further injury
c. Will follow varied schedules of activity and rest
d. Dernonstrate decreased anxiety levels
140.
Prior to the fall accident , the wife noted Mr. Lazos changes in sleeping and eating habits getting
irritated easily and lacking interest in usually pleasurable things. The nurse infers correctly a possibility
of the presence of
a. Obsession

b. Anxiety
c. Depression
d. Social isolation
141.
All of the following nursing interventions are appropriate for Mr. Lazo EXCEPT:
a. Use calendars , sign and labels as needed
b. Offer activities that challenge creativity and innovation
c. Encourage clients to talk about past events
d. Limit client decision making
142. Mr. Lazo became agitated and combative when the nurse approached him to help with morning care.
The nurse utilizes validation technique by saying.
a. You need something to calm you down.
b. You need restraint for safety.
c. It is time to get dressed.
d. You seem upset.
e.
f.

Potrebbero piacerti anche